Saturday, August 16, 2014

Passed ASWB test

I passed !!!!!!!!! Yes...I did it.......... My God, All glory be to you.................. am so glad...............I thank Blessed Mother and my dear Guardian Angel for all that they did to me to get me through this......... ummmaahhhh

Friday, August 15, 2014

Day 14, 15 key concepts and final revision

Few points to add on at the end of the prep, use this the day before you go to test ....
  • while answering the primary thing is to read all the answers put them in order, read specifier the question asks and then choose the answer
  • concerns with safety is primary issue for the worker. ethics is one of the reason which we look into for  guidelines in saving clients and making clinical decisions. issues of child abuse, domestic violence, homicide threat, suicide threat. all are important issues where safety and reporting you must be sure of . remember even "neglect" can trigger child abuse report. once you see child abuse (physical, emotional, sexual, neglect) look out for medical evaluation, reporting as key answers. once u see domestic violence look for key words like safety plan. suicide reporting overlaps confidentiality so is homicide threat. the main attitude we need to carry here is "safety of client" being priority. also abuse and neglect of elderly- report to adult protection services, note down key words for self neglect, financial abuse, using restraints, social isolation and abandonment.
  • also reporting issues varies with state...so look for choice "according to jurisdiction" procedure
  •  suicide and homicide...clarify threat. and if there's plan you have to notify victim and officials and confidentiality overrides here
  • suicide...look for key words hopelessness, history of suicide attempts, if assessment hasn't been done, think of assessing the risk
  • with respect to confidentiality be very careful. even acknowledging or getting to know someone is coming to treatment to you is a breach of confidentiality.  
  • study dsm in terms of how they differ in that class of disorders. childhood disorders differentials and other disorders
  • browse through timeline differences for various disorders. try to note down timeline mentioned in question and answer accordingly also note down expression of symptoms by various kind of population..like how in children, how its manifested in elderly
  • do not look too much in to medications, just brush through much frequent ones used when you are going through them before test
  • note down if its positive or negative questioning style... like which is "NOT" ... you might lose this if you dont pay attention
  • always rule out medical condition, substance use for clinical decision making
  • someone died being mentioned in question.... think about grief
  • persistent belief of having serious illness despite no evidence to prove - hypochondriasis
  • psychological condition converted to physical manifestation symptom - conversion disorder
  • sensate focus intervention -intervention in sexual intimacy issues without intercourse
  • get to know the classification of disorders, you might be tested on this... eg; which of these is not paraphilias
  • borderline PD- unstable relationship, social worker need to keep firm boundaries with clients..always discuss feelings in therapeutic relationship context with clients, calling in between appointments as key words
  • schizoid PD-  loner, like being alone
  • avoidant PD-  lonely, but want to be social
  • developmental issues question, always look for misinterpretation of parents, culture issues, immigration issue, psycho education for parents as key words 
  • "life review " - intervention with elderly, get information about their life, how they lived, attained, in last stage of erickson's theory, you get to involve them much better in intervention.
  • no secrets policy in couples issue to be ensured in couples intervention and treat the common issue they came for and not individual intervention unless domestic violence issue comes up
  • DABDA - denial, anger, bargaining, depression, acceptance (kubler ross stages of grief)
  • TAIIIIGI - trust vs mistrust (birth - 18 month), autonomy vs shame /doubt(18 months- 3 yr), initiative vs guilt (3-6), industry vs inferiority (school 6-12), identity vs role confusion (adolescent developing identity 12-20), intimacy vs isolation (early adulthood 20-40), generativity vs stagnation (creating next generation, parenting 40-65), ego integrity vs despair (meaning  and accomplishment in life,late adult - 65)
  • oedipus complex- castration anxiety, electra complex- penis envy
  • Seperation anxiety  begins 6-8 months, peaks in intensity 14-18 m, until about 2 and dimnishes
  • ASS - Margret Mahlr's staes of development A - autistic (new born- 1 month) S - Symbiotic (fusion with mother, child think its one with mother and not a separate individual 4- 6 wees - 5 month) ,  S- Seperation individuation (Hatching, Practicing, Rapprochement (beginning, crisis, solution))
  • SPCF - Jean Piaget's theory of cognitive development . S- sensorimotor (birth -2) object permanance , P- Pre operational (2-6) ego centrism , concrete operations stage (6-12) conservation , F- formal operations (12 and above) abstract thinking and concentration
  • PSBSS - Maslow's heirarchy of needs P- physiological, S-  safety, B- belonging, S- self esteem, S- self actualization
  • PCP - Lawrence kholberg's theory of moral development. stages include Preconventional morality(2-4,obedience and punishment orientation, 4-7 self interest orientation) , Conventional morality(7-10 social conformity, 10-12 law and order orientation) and post conventional morality(teens- social contract orientation, adult- universal ethics orientation)
  • AAIPIU- Authoritative parenting(displaying rational control, warmth and responsiveness, promoting independance-assertive, self confident, achievement oriented children), authoritarian parenting(high control and little warmth, hostile, rebellious withdrawn children with low self esteem and poor achievement), indulgent - permissive parenting(warm and caring but provide little control and make few demands, children are indulgent, frustrated, low in achievement), indulgent - uninvolved parenting(less time and effort in parenting, these children have low self esteem, delinquent, rebellious, impulsive, moody, aggressive) - Diana Baumrind parenting styles
  • MSE - mental status exam - different domains- appearance, attitude (resistant, eager, over optimistic), behavior, mood and affect (flat, elevated, connected or disconnected presence), speech, thought process, content of thoughts, perceptions, cognition, insight, judgement
  • in issue of terminating client, we can never abandon the client, common response to termination is feeling of being fearful to able to handle issues without therapist, crisis around termination doesn require termination, reactions are common.
  • Medications important for social workers knowledge include lithium - associated with Bipolar Disorder , has adverse side effects.
  • Typical antipsychotics (Psychotic Disorders) • Chlorpromazine (Thorazine) • Haloperidol (Haldol)
  • Atypical antipsychotics (Psychotic Disorders) • Risperidone (Risperdal) • Olanzapine  (Zyprexa)  Quetiapine (Seroquel) • Ziprasidone (Geodon) • Aripiprazole (Abilify) 
  • SSRI’s (Depression/Anxiety) • Fluoxetine (Prozac) • Citalopram (Celexa) • Sertraline (Zoloft)   Paroxetine (Paxil) • Escitalopram (Lexapro) Mood Stablizers (Bipolar) • Lithium • Divalproex Sodium (Depakote) • Carbamazepine (Tegretol) • Lamotragine (Lamictal) • Oxcarbazepine (Trileptal) Benzodiazepines (Extreme Anxiety/Panic) • Clonazepam (Klonopin) • Lorazepam (Ativan) • Alprazolam (Xanax) 
  • Stimulants (ADHD) • Methylphenidate  (Ritalin, Metadate, Concerta, Daytrana) • Amphetamine (Adderall) • Dextroamphetamine (Dexedrine, Dextrostat
  • get to know the theories. its not tuf. associate the theories with the concepts and developers. some key theories and key words include                                                                                                      behaviorist theory - operant conditioning, classical conditioning , positive and negative reinforcement, punishment,                                                                                                                  CBT - thinking about people and problem,                                                                                        structural family therapy (salvador minuchin)-  system approach -  key concepts- boundries, heirarchy, subsystems, childrens problems are due to boundary between parents and between parents and children, mimesis, structural mapping, raise intensity so system change                                           task centered therapy/ social work practice - 6-8 weeks short term problem solving, concrete plan to intervene, goal oriented.                                                                                                            psychodynamic -  freud's approach, unconscious, defense mechanisms - primitive (denial, regression, projection, reaction formation,  ), mature(displacement,) and advanced defense mechanisms(sublimation - unacceptable feeling is made in a way acceptable, like sarcasm and kidding around, getting hard workout to deal with unwanted sexual impulse), insight and understanding into past is another key word for identifying with psychodynamic approach                                                 feminist the and social learning th-                                                                                                 problem solving model          
       Bowen family therapy -  key concepts differentiation of self, triangulation, multi generational       
        transmission process, emotional reactivity      
  •       Experiential family therapy -  carl whittaker, virginia satir- key concepts - honest emotion, supress emotion, mystification, blaming, placating, sculpting, conjoint family drawing, choreography, role playing                  
  • never accept bartering, dual relationships, these always come in way of quality of treatment and therapeutic relationship
  • alderian psychotherapy appropriate for issues of goal direction, existential psychotherapy for high functioning individuals, 
  • boundaries is another important issue. here comes transference (client rings up feelings regard to worker) and counter transference (worker affected by client) issues, dual relationship, overlap in relationship, be clear on value differences and never let it in therapy process, in terms of dilemma go for supervision
  • go through ethics before the day of the exam. its very important which guides in your clinical diagnosis or decision making
  • what's privilege ? term related to confidentiality in terms of legal context. where the social worker need not reply to the subpoenas other than from judge. however privilege is waived in number of other conditions like self exposed, suing social worker
  • concepts about group, most important is stages of group formation. storming, forming , norming- irving yaloms theory, boston model -  , group process, 
  • team work -  as a part of multi disciplinary team whats the role for social worker in terms of  coordinating services of client
  • advocacy- empowering client on getting services they are entitled to access them themselves. start where the client is. anything about program development has to do with clients need , program evaluation and methods are another area where question is asked. methods of evaluation, best practices, terms associated
  • cultural diversity is another important area. navajo community, asian clients,african american, hispanic is few areas included in test. issues of immigration, cultural shock, eye contact, aggressive talk, are certain areas we could be tested.
  • Some sclaes/test commonly used: Standford binet intelligence scale, wechsler adult intelligence scale, toni- for multicultural clients, MMPI - minnesota multiphasic personality inventory, 16 personality factor questionnaire, Meyers brigs type indicator, edwards personal preference schedule, DAD, dyadic adjustment scale- for marital adjustment, FES family environment scales, the camberwell family interview, assessment tools for children: childhood autism rating scale, children's depression inventory, Draw a person test, kinetic family drawing, vineland adaptive behavior scales, AUI- alcohol use inventory, 

St. Aquinas prayer

Come, Holy Spirit, Divine Creator, true source of light and fountain of wisdom! Pour forth your brilliance upon my dense intellect, dissipate the darkness which covers me, that of sin and of ignorance. Grant me a penetrating mind to understand, a retentive memory, method and ease in learning, the lucidity to comprehend, and abundant grace in expressing myself. Guide the beginning of my work, direct its progress, and bring it to successful completion. This I ask through Jesus Christ, true God and true man, living and reigning with You and the Father, forever and ever.

Amen.

Day, 13

Practice test practice test practice test.......... there is nothing more you should do for the last 3 days...at least last 2 days...focus on remaining relaxed and control your anxiety. proper diet and sleep very important. Because it's going to affect the way you think during the exam.

Try to have a look at the notes and do not do any kind of referencing job these days. try to stay positive and confident. Its not a big thing. Take several deep breaths whenever you feel the tension building up in the system. 

ASWB practice test is very much recommended so you don't go there unprepared to face the software. it eases you to know the virtual environment you are going to face. Take the exam and review the questions.

try to take a break during the exam, drink juice or snack...you can't take the whole exam in 4 hr at a time. Your brain needs nutrients and atleast water to keep your brain fresh to think through all 4 hr...so have a banana.. and try to take a break if see yourself anxious, unfocused because u r getting tired. 

Do not memorize anything hard, just read through as a story where you understand and involve in the matter you are going through.

Have enough sleep and take public transport to the examination facility and above all i strongly recommend PRAYER :-) it works wonders !!!! if someone wants to pray for you, let them do it, do not turn down the invitation, anyway its not going to harm you.......

Tomorrow is my test........ Will update the blog on more tips and notes. 

All the Best and God Bless !!!!!!!!!!!!

Thursday, August 14, 2014

more strategies

1.  Always try to teach others what you have studied, specially those who do not know about your field and encourage them to question or challenge you. You'll be surprised by the different ways you get the questions and the different ways you try to find the answer for it finally to know you have mastered the topic. I always do this with my poor husband. So all those out there who want to take revenge, get your spouse to help you in your career by listening to you for 30 min (a better way to build a relationship too) or if there's no one out there...........use your doll...sounds crazy huh...try out

2.  Always take part in group discussion. theres plenty out in Facebook, skype  and other social media. this helps more than you think, but forget to use the idea during the end of the test. Do it while you are preparing.

3.  Always give the last 3 days for rest. You never know how your mishandled anxiety can treat you. Am suffering with unexplained body aches and fatigue and too tired to even revise my stuff. And there's just one day left for the exam. So never plan till last minute and never push yourself through the final days.

4.  always look for key words and mark them down or scribble them down. Key words are most important. They are specifiers like "should, best, first, next, and in the answers (acknowledge, validate, explore, clarify, assess, refer, etc.

When approaching questions: 

(1) Acknowledge feelings first if given the option (you may also want to validate this is similar to acknowledge, validate what they are FEELING only)
Remember acknowledge simply means you are aware and state what you see
Validation means you can understand how they feel. Sometimes we acknowledge but not validate.
(2) Explore the situation before you clarify
(3) Identify before  clarify, and when you clarify make sure the client is with you and it is mutual so you can be sure they understand
(4) Get client's perspective first (e.g. the client asks the question, what do you think, get client to say what he/she thinks before you give your suggestions)
(5) For the group related questions always bring  it back to the group (unless danger to self or others, mandatory reporting, etc.)
(6) If it is very sensitive in nature the topic the client wants to discuss and you are concerned about the client sharing such sensitive information offer to discuss individually any sensitive topics that may cause harm and/or embarrassment when shared in a group setting. 


Below, is a question and an example on how to put the key words on scrap paper.

Mr.jones is having dialysis and has missed his last two treatments.  In addition, his wife is in poor health and he is the primary caregiver for her.  In discussing his situation, he sees it as hopeless and shows other signs of depression. The client states he wants to quit dialysis and die. A multi-disciplinary team meeting is called.  The social worker should advocate for:

A. letting the client (mr.jones) exercise his right to die
B. evaluating the client for depression
C. suggest that he see a therapist for the depression
D. tell the physician to increase his medicine

Answer B, from these choices is evaluate for depression.  The key words in these answers are: 
letting
evaluating
suggest
tell physician

I would evaluate before I did any of the others.  This man needs to be evaluated for depression he will also need to be evaluated for suicide potential and the answer that says evaluation gets closest to that. 
The team needs this information to make an informed decision on how
to best assist this client.



Wednesday, August 13, 2014

Day 12

Brush through certain concepts to read further and recall necessary test information


  1. What is peer review ?
  2. Jeff vs redmond case vs social work decision making
  3. confidentiality and insurance company
  4. what is vicarious liability?
  5. timeline difference between MDD and Adjustment disorder with depressed mood and acute stress disorder
  6. what is type I error in statistical research?
  7. Albert R roberts 7 stage crisis intervention model?
  8. difference between conduct disorder and disruptive behavior disorder
  9. what are 4 classic "A" s of schizophrenic diagnosis
  10. what is Munchausen's Syndrome.?
  11. what is sancter's disease
  12. what is narcissistic injury?
  13. What does the term "overhead" mean in planning program
The hard lesson learnt is....give more than 10 days before the exam to do the practice test. Then you'll know you need additional 10 days for the set of concepts you faced for first time. 

Tuesday, August 12, 2014

Day 11


1. Anxiety Disorder 
2. Mood Disorder
3. Practice test and additional concepts

Defense mechanisms and disorders - frequently asked question
http://charlessamenowmd.com/wp-content/uploads/2012/06/Defense-mechanisms-and-Personality-Disorders.pdf

Study therapies in key terms too. like distinct goals for each therapy, key words in each therapy, practice identifying therapy using its key concepts, effective therapy for respective disorders etc. follow example below. you may device one by yourself which is much effective than using those done by others.

Rapid change situation -                          Crisis intervention
children -                                           evaluation by play therapy
depression -                                         cognitive therapy, evaluate for suicide
suicide -                                             look out for plan
threat, homicide -                        look out for plan, inform authorities
domestic violence                              safety plan in absence of abusive partner
intervention with children -                   first medical evaluation
borderline -                                          DBT
emphasis on client  -                               rogerian
grief-                                        kubler ross
fatigue -                                          physical examination exclusion
medical condition -                                                 refer
minor -                                             jurisdictional requirement
religion -                                                 belief and values and their meaning
substance abuse -                              difference in dependence and abuse, support group
seperation individuation -                   Mahlr
develop -                                       frued, erickson , piaget stages of development
personality disorders -                  defenses behind
irrational belief -                             REBT

building your own helps you not only with recall but also confidence.

Monday, August 11, 2014

Anxiety Disorders

Disorders in this section include: 


Panic Attack
Agoraphobia
Panic disorder without Agoraphobia
Agoraphobia without history of panic disorder

Specific phobia : Persistent fear of specific object or situation 5 types animal type, natural environment type, blood injection injury type, situational type and other

Social Phobia : fear of social performance situation, exposure to unfamiliar people and scrutiny by others

OCD (Obsessive compulsive disorder) = Obsessions (recurrent persistent intrusive thoughts that cause marked anxiety) + Compulsions(repetitive behaviors or acts aimed at preventing or reducing the distress)

PTSD (Post traumatic stress disorder) = exposure to traumatic event + persistently reexperiencing the event + avoidance of stimuli associated with trauma + increased arousal

ASD (acute stress disorder) symptoms occuring within 1 month of extreme stressor and PTSD requires symptoms present for more than 1 month. symptoms lasts from 2 days to 4 weeks and occurs within 4 weeks of traumatic event

GAD (generalized anxiety disorder) - excessive anxiety and worry for at least 6 weeks about a number of issues with difficulty in controlling the worry








 

 

  


  

To be better prepared for test try to check if you are confident answering these questions and clearly know the differentials

  1. Difference between panic disorder with agoraphobia and without agoraphobia vs specific phobia
  2. Difference between PTSD and ASD
  3. PTSD vs Adjustment disorder vs ASD

Mood Disorders



Bipolar II
 

Bipolar Disorder


Mood Disorders include 4 types:


  1. Depressive disorders : Major Depressive Disorder (MDD) , Dysthymia , MDD NOS
  2. Bipolar Disorders -  Bipolar I, Bipolar II, cyclothymia, Bipolar NOS
  3. MD due to general med condition 
  4. MD due to substance
The building blocks are 4 kinds of mood episodes
  1. Major depressive episode 2wk + depressed mood or loss of interest or pleasure
  2. Manic episode - abnormally elevated expansive or irritable mood for at least 1 wk
  3. Mixed episode -  both criteria for manic and mixed episode for 1 week
  4. Hypomanic episode - less severe manic symptoms, not too low for depressive symptom, upto 4 days


 



Different ways you might be tested regarding Mood Disorders: (take this as an exercise and find the right answer)

1. differences between Major Depressive "episode" (not disorder, watch out for these specifier) and Adjustment disorder with depressed mood  and Breavement

2. manic episode vs hypomanic episode

3. difference between MDD and Dysthymic Disorder

4. difference between schizoaffective disorder and MDD with psychotic features

5. differential diagnosis of dementia and MDD in elderly

6. Bipolar I vs MDD vs dysthymia


to view :



Refer : DSM

Day 10

Took ASWB practice test..... scored 96/170...passing score is above 100/150...so the most important thing about preparing for exam is to finish all your study before 2 weeks and then start doing only practice test for next two weeks. You'll  be surprised to find out how much you missed on the way to preparation and the trend to conquer the exam.

1. you will be confused in finding answer for something you already know. eg: you might know the developmental stages in erickson's theory but if asked what is the third stage of the theory and given choices will confuse you in the best way. So revising your factual information after taking each single test will help you retain concepts and be confident of direct answers you can score for sure in the exam.

2. ethics is most important in leading you to decide choosing best answer while answering each question. revise ethics every time. You will not regret.

3. Note down the answers and read on the topic. It helps more than memorizing

4. always spend time on rationale. this will help you to see questions from right perspective.

Concepts to brush through after taking the first set of practice exams:

  1. Navajo community and social work issues with service delivery?
  2. supervision
  3. differences in concepts defense mechanism and  transference and counter transference
  4. group types and selection criteria
  5. reporting procedure in abuse
  6. communication in social work
  7. program evaluation steps and types
  8. social policy analysis steps
  9. working in a team
  10. differential diagnosis of anxiety and mood disorders                                                                                         
  11. abuser victim relationship
  12. difference in withdrawal symptoms of various drugs                                                                                                                                                                                                                                                        
  13. what is privileged communication : can be broken at certain exceptional situations ref : the practice of social work by charles zastrow pg: 44                                                                                                                                                                                                                                                                               
  14. confidentiality related to HIV : name associated testing and anonymous testing : the former involves identity and later involves no identification using a code word or number where names are neither mentioned nor recorded , reporting issues, psychoeducation. same book above . pg : 46... likely to be tested                                                                                                                                                                                                        
  15. State regulations pertaining to clinical records (NJ) : http://www.njconsumeraffairs.gov/chapters/Chapter%2044G%20State%20Board%20of%20Social%20Work%20Examiners.pdf this link also contains several other state regulated activities for Social Workers, this can help those in NJ. Others try to find similar document from your respective state websites.also include: age for minority in various state                                                                                                                                                                                                                                                                                                 
The social worker shall retain the permanent client record for at least seven years from the date of the last entry, unless otherwise provided by law, or in the case of a minor, until age 25.


This should give you an idea of how you can approach test in one way.

Sunday, August 10, 2014

Day 9

Practice Questions and more concepts....

Tip :

When practicing the questions, it is very important to review them in the context of what you missed in it. There are some subtle details like, pointers, age, onset, specifiers which will retain in memory only after repeated exposure to that information. try something lie below which i developed after doing a 50 practice question set. say i received 30/50, am going to point out, find out, explore what i missed in those 30. You may do it for your review so you get familiar with details about the subject.

1. Failure to thrive syndrome :
Nonorganic FTT is often a complex of disordered interaction between a child and caregiver. In some cases, the psychologic basis of nonorganic FTT appears similar to that of "hospitalism," a syndrome observed in infants who have depression secondary to stimulus deprivation. The unstimulated child becomes depressed, apathetic, and ultimately anorexic. Stimulation may be lacking because the caregiver is depressed or apathetic, has poor parenting skills, is anxious about or unfulfilled by the caregiving role, feels hostile toward the child, or is responding to real or perceived external stresses (eg. demands of other children in large or chaotic families, marital dysfunction, a significant loss, financial difficulties).
Poor caregiving does not fully account for all cases of nonorganic FTT. The child's temperament, capacities, and responses help shape caregiver nurturance patterns. Common scenarios involve parent-child mismatches, in which the child's demands, although not pathologic, cannot be adequately met by the parents, who might, however, do well with a child who has different needs or even with the same child under different circumstances.
2. Parenting styles and outcome :
3. What does and doesn MSE evaluate : does not evaluate social relationships
4. core values of social work ?
Service, social justice, dignity and worth of the person, importance of human relationships, integrity and competence. they might ask a question like this 
.All the following are social work core values EXCEPT:
A Self Determination
B Social Justice
C Service
D Integrity

5. What is transtheoritical model ?

The transtheoretical model of behavior change assesses an individual's readiness to act on a new healthier behavior, and provides strategies, or processes of change to guide the individual through the stages of change to Action and Maintenance. 


6. Boston Model of Group Development


Saturday, August 9, 2014

Day 8

Practice questions and Differentials for Mood disorders, anxiety disorders and psychotic disorders....

Tip : try to help as many people as you can when you prepare for you exam. Discussions and sharing of information throws light into different ways of looking at a question you might miss from your perspective.

Always have a day or 2 in reserve to catch up what u miss in your plan days...nice sleep and proper rest is very important in your preparation. Without these two am missing so  much time and energy.

Friday, August 8, 2014

170 Practice questions



1. A 16 year old teenage boy was referred to therapy to address interpersonal problems and aggressive behaviors. He has been breaking his curfews and has a history of truancy. He was recently suspended for fighting. As the social worker working with this client you would FIRST:
A Discuss with the teenager the consequences of his behavior
B Explore family Dynamics
C Demonstrate candor in order to build a rapport with teenager 
D Assess for a diagnosis of conduct disorder


2. A 40 year old woman just gave birth to her first child. Client sought therapy to address her own history of child maltreatment. The social worker observes that the client’s newborn seems to be experiencing insecure attachment. As a social worker treating this client the Most Appropriate modality to treat this client is: 
A Gestalt therapy to deepen the awareness of self.
B Problem Solving Therapy
C Psychoanalytic Therapy.
D Motivational Interviewing.


3. Cognitive Behavioral Therapy is most appropriate for treating all of the following disorders EXCEPT:
A Mood Disorder
BPervasive Developmental
C Eating Disorder
D Substance Abuse Disorder


 4.  A client is 5 year old boy who identifies with his father. He enjoys spending time with his father and does things like playing soccer, watching superhero movies and playing video games with him. Mother is concerned that client prefers his father's company.  As a psychoanalytic social worker you would assume that the child is Most Likely:
A In the Anal phase and feels overly confident in doing things like his father
B Individuated from the mother
C Is in the Phallic stage and Feels Castrized by the father  because he would like to be  intimate with mother
D Has overcome the Oeipidal conflict and no longer wants to have sex with mother and identifies due to development of the superego
 

5.All the following are social work core values EXCEPT:
A Self Determination
B Social Justice
C Service
D Integrity


6. Ms. Jones is a social worker in a hospital setting, she is informed by a HIV positive patient that he is having unprotected sex with males. As a ethical social worker Ms. Jones should FIRST:
A Tell the patient that she is obligated report it to the health department
B Inform patient that if he continues to have unprotected sex he would be discharged from the clinic.
C Respect the patient’s privacy and self determination
D Provide patient with psycho-education about safe sex.



7.  A man who attended several marriage counseling sessions with his wife, stated in one session that he would kill his wife if she left him.  The social worker is aware that the wife has filed for divorce.  A ethical social worker should FIRST:
A Explore the seriousness of husband’s statement.
B Report the threat to authorities to prevent harm to the wife
C Conduct a mental status exam on the husband at the next session.
D Read the code of ethics before discussing issue with couple in the next session.


8. Jill is a school social in a High School, in which a large number of the students are from Central America.  Jill was hired to provide individual and group therapy to students experiencing emotional and behavioral problems. Jill was called by the 10th grade teacher to assist her in controlling the class.  The teacher believes that the language barrier is affecting her communication with students. The social worker Most Appropriate action is to:
A Explain to the teacher that this poses a role conflict
B Assess whether the problem stems from racism or inflexibility
C Observe the class and assess whether the problem is caused by the students or the teacher.
D Schedule a meeting with the principal and the teacher regarding this issue.


9.  A social worker is conducting an intake with a client a 17 year old male, who has a long history of impulsive behaviors, racing thoughts, inadequate sleep, and stealing. Client reported that since being incarcerated 3 times and hospitalized 5 times he is now stable. Client reports that he wants to get his High School diploma so that he can become a doctor but he fails to follow through on any assignments he starts and reports periods of depressed mood. The Most Appropriate diagnosis for this client is
A Dysthymic Disorder
B Schizzoaffective
C Borderline Personality
D Bipolar



10.You are a school social worker treating a 12year old female who is experiencing social anxiety. Client reported that when she goes to school she feels everyone is talking about her. Client reported that she experiences panic attacks when she enters the school. Client leaves school early everyday because she is unable to sit still in class and is worried she will experience a panic attack. Client reported that the panic attacks started when she was bullied in the second grade. Client’s parents don’t believe client has a problem with anxiety and feels that client prefers to stay home to play video games and watch TV. The Most Appropriate action is to:
A Continue using systematic desenzitization to assist client with reducing her anxiety at school
B Teach client how to soothe self using relaxation and meditation techniques
C Join with the parents and assist parents in developing a behavioral plan
D Schedule client for comprehensive evaluation through the school in order to assess the need for services and possible medication management for the child.


11. A client diagnosed with Post Traumatic Stress Disorder experiences panic attacks and flashbacks when she drives on the highway. Client reports that she stops on highway 4 to 5 times before reaching her destination. Client is worried that she will lose her job due to her lateness. What psychotropic medication would be used treat this client?
A Anxiolytic.
B. Antidepressant.
C. Neuropleptics.
D. serotonin reuptake inhibitors.


12. You are treating a child who is exhibiting disruptive behavior, speech difficulties and excessive neediness at school. In speaking with the mother, the social worker learns that client's mother separated from client’s father 3 months ago, she uses a wheelchair and has no family support. The Most Appropriate action is to:
A Engage client,father and mother in the next family therapy session.
B Assess for the need of increased sessions for the family.
C Explore whether family stressors are impacting the client and broker for support services.
D Suggest a comprehensive school evaluation for the client.


13. A social worker has been seeing a couple in couples therapy for the past month. The social worker recently learns that the reason for the husband's failed marriages is because he has sex with several women. The husband is remains honest to his current wife about his extramarital affairs. The wife is unhappy that her husband has sex with other women but has decided not to divorce her husband. The social worker is Roman Catholic and holds different values about marriage and extramarital relations. The Most Appropriate action for the social worker is to:
A Seek ongoing supervision about countertransference issues and transfer client to another social worker if value conflict persists.
B Refer the husband to a sex therapist.
C Social Worker should keep her own values to herself so that she will not influence therapy.
D Examine the problems that the couples are presenting.


14. A social worker treating a client with Cognitive Behavior Therapy. Client reports feeling guilty when he steals from his elderly mother. The social worker explores clients triggers and uses cognitive restructuring to assist client in feeling less guilt. The social worker and the client realizes that client's stealing has not reduced. What is the best interpretation of this situation?
A Client needs a higher level of care.
B Client has been stealing since he was a child and its too late for him to stop stealing.
C. Client concludes that social worker considers such behavior as acceptable.
D Client wants the social worker to report him to authorities.


15. What psychiatric condition describes a person with a lifelong pattern of flattened affect and social isolation?
A  Antisocial Personality Disorder
B Schizoid Personality Disorder
C.Dysthymic Disorder
D Schizotypal Personality Disorder



16.  A 23 year old female woman presents during intake with provocative behavior and excessive emotionality.  Client reported during the intake she reported that her image is very important to her and that she blames her mother for all the mistakes she has made in her life. The Most Likely diagnosis is:
A Paranoid Personality Disorder
B Generalized Anxiety Disorder
C. Dependent Personality Disorder
D Histrionic Personality Disorder

17. An elderly man is mourning the loss of his wife and is seeking treatment to address loss. What is the first step the social worker should take?
A.  Help the client work through the process of grief
B. Assist client in utilizing the empty chair technique; which is talk to chair as if the deceased person were actually sitting there.
C Teach client coping strategies
D Refer client to a grief support group for the elderly.


18. All of the following are reasons why Cultural Diversity is an important consideration in social work ethical decision making, EXCEPT:
A When services are provided in the language appropriate to the client it strengthens the  therapeutic relationship
B Social workers’ understanding of the role of culture in the helping process.
C It allows social workers to advocate for social policies and programs that affect diverse client
populations,
D The United States is constantly undergoing major demographic changes.


19.Trichotillomania is classified in the DSM IV as;
A. Impulse Control Disorder
B Anxiety Disorder
C Mood Disorder
D Adjustment Disorder.


20.  A forty five year old women reports experiencing pain in her neck for the past 6 months since getting divorced.  Client denied experiencing medical problems. The Most Likely diagnosis is;
A Somatization Disorder
B. Dyspareunia
C Pain Disorder with psychological factors (Chronic)
D Conversion Disorder




21. The best predictor of suicidal behavior is
 
A

Feelings of hopelessness. 
B

severity of somatic complaints. 
C

social withdrawal. 
D

impulsivity. 


22. Schizophrenia is typically characterized by profound disturbances in thinking, affect, perception, and psychomotor behavior. It is generally believed that a poor prognosis for schizophrenia is associated with all of the following EXCEPT
 
A

onset in late adolescence. 
B

acute onset of symptoms. 
C

lack of emotional responsivity. 
D

obsessive-compulsive symptoms. 



23. "Depersonalization" is characterized by
 
A

disorientation and lack of ego functions. 
B

free-floating anxiety. 
C

flooding of the mind with persistent, uncontrollable thoughts. 
D

extreme fear and avoidance. 


24. Two teenage boys have been experimenting with drugs and sexual behavior. They have been fastening plastic bags over their heads. What would you suspect?
 
A

autoerotic asphyxia 
B

drug abuse 
C

suicidal behavior 
D

deviant behavior 


25. A client diagnosed with a dual disorder (alcohol dependence and bipolar I disorder) is being discharged from a residential treatment facility and will be returning home to live with his family. What is the primary issue?
 
A

intervening with the client's family 
B

referring the client to a 12-step group 
C

referring the client to vocational rehabilitation services 
D

determining how the client will support himself 


26. Which family characteristic is most likely to be associated with the diagnosis "failure to thrive"?
 
A

enmeshment 
B

disengagement 
C

low SES 
D

parent-child interaction problems 



27. A child has an IQ of 30. His parents ask you what they can expect their child to be able to do as an adult. What skills do you describe?
 
A

communication and social skills, academic skills up the sixth grade level, vocational skills for minimum self-support, community living 
B

communication skills, basic vocational and self-care skills, ability to travel independently in familiar places 
C

basic self-care skills, ability to talk and read "survival" words, ability to perform simple tasks in a closely supervised setting 
D

academic skills up to the second grade level, adaptation to community living in a supervised setting 


28. In terms of interpersonal relationships, a person with schizotypal personality disorder is most likely
 
A

to seem indifferent to opportunities to develop close relationships and to not derive pleasure from personal contacts. 
B

to be preoccupied with doubts about the loyalty and trustworthiness of friends. 
C

to desire close personal relationships but to avoid them because of fears related to criticism and rejection. 
D

to express unhappiness about a lack of relationships but to act in ways that suggest a lack of interest in intimate contacts. 


29. Somatoform disorders are
 
A

physical illnesses. 
B

emotional illnesses. 
C

emotional and physical illnesses. 
D

a form of psychosis.  


30. Behavioral assessment is distinguished from traditional assessment in several ways. Which of the following is NOT a characteristic associated with behavioral assessment?
 
A

Focuses on directly observable behaviors and environmental factors. 
B

Views outward behaviors as signs of an individual's underlying characteristics. 
C

Involves assessing behaviors repeatedly throughout the course of the intervention. 
D

Involves assessing specific (rather than global) aspects of the individual's behaviors. 



31.Which of the following is required for a diagnosis of encopresis, according to DSM-IV-TR?
A

the symptomatic behavior must be involuntary
B

the onset of the disorder must be preceded by a period of fecal continence
C

there must be at least one symptomatic event per week for at least one month
D

the child’s chronological or mental age must be at least 4 years


32. In the initial session with an elderly patient who displays signs of depression, it is most important to
 
A

refer for psychological testing. 
B

take a social history. 
C

explore the patient's support systems. 
D

refer to a neurologist. 




33. Which of the following is the most accurate indicator of a client's level of motivation to change in therapy?
 
A

What led him to come to treatment. 
B

His coping skills. 
C

His body language. 
D

His willingness to agree to therapeutic goals.  


34. With a client, role-playing is useful for all of the following purposes, EXCEPT:
A

preparing her to engage in a new behavior
B

identifying the source of her social role ambiguity
C

enhancing her understanding of others
D

enhancing her self-awareness


35. A 60 year old women comes into therapy hesitantly. She describes a life that is "empty," and a  feeling that she too is "empty." History reveals that she is exceedingly uncomfortable in situations where she is not the center of attention. Her behavior is not age-appropriate, and she is known for sexually seductive and provocative behavior. Her emotions are shallow and she depends on her physical appearance to draw attention to herself. Her speech is excessively impressionistic and lacking in detail. She is theatrical and easily influenced by others. She considers her relationships to be more intimate than they are. Based on the above information which of the following DSMIV TR diagnoses seems [MOST] appropriate?

A. Dependent Personality Disorder
B. Histrionic Personality Disorder
 C. Obsessive-Compulsive Personality Disorder
 D. Borderline Personality Disorder
 













36. Parents adopting an authoritative style are most likely to produce children who are
 
A

dependent, passive, and submissive. 
B

disobedient, aggressive, and rebellious. 
C

independent, self-confident, and self-controlled. 
D

moody, creative, and independent. 



37. A social worker is in the first face-to-face meeting with an involuntary client. The client has been mandated by the court to seek professional help. The social worker should FIRST:
A

address the client’s negative feelings about coming to therapy
B

 discuss with the client what will happen if he does not cooperate with court’s requirements
C

attempt to identify a treatment goal that the client is motivated to work on
D

 share with the client the information she has about why he has come in and invite him to tell his side of the story


38. A 23-year-old client is distressed over a decision he is trying to make. The client is Catholic and believes that premarital sex is wrong. He goes to church regularly and his religious faith is important to him. His girlfriend of two years, whom he loves very much, has begun to pressure him for sex. He has explained his position to her, but she keeps pointing out that other couples their age have sex. She has almost convinced him that his views are outdated. He is now completely confused. The social worker’s MOST appropriate intervention is to:
A find out whether the client has other problems in his relationship with his girlfriend

B help the client understand the source of his confusion and monitor for information suggesting that he was sexually abused during childhood

C reassure the client that his confusion is normal since his views are, in a sense, “outdated”

D encourage the client to talk to his priest if he hasn’t done so already



39. A social worker using solution-focused therapy with a client might do which of the following in the first session?
A ask the client to describe what he thinks is preventing him from finding solutions to his problem

B work with the client to identify and describe therapy goals in concrete, positive terms

C evaluate the client’s problem-solving skills and deficits

D identify the interrelated causes of the client’s problem



40. According to Erikson, at age two, the primary task of psychosocial development is to develop ___________, and failure to successfully accomplish this task leads to ___________.
 
A

an ego identity; identity diffusion. 
B

a capacity for industry; a sense of inferiority. 
C

a sense of basic trust; suspicion and mistrust. 
D

a sense of autonomy; doubt and shame. 


41. The Social Worker has seen a couple for marital therapy. Six months after leaving therapy, the couple decides to divorce. There are two children, age six and four, whose custody is to be determined. The husband contacts the Social Worker and requests that she testify as to the wife's alcoholism at the custody hearings. The [MOST] appropriate response for the Social Worker would be:
        
 A  to agree to his request since mother is clearly unfit.
 B  to contact mother and suggest that she and her husband return for couples work.
 C  to suggest that the Court contract a child custody specialist for an evaluation.
 D  to recommend that the children be invited to testify as to their experiences with mother.













42. A single woman with two toddlers comes in for a first appointment to see a social worker. When you made the appointment, her stated reason for wanting counseling was to deal with her anger. You do not usually inform clients during the initial phone call about limits to confidentiality, preferring to cover that in person the first session. At the beginning of the appointment, the woman states she needs help with her anger. She tells you she has recently become so angry at her 3 year old that she “spanked him with a hairbrush” much harder than she had intended, leaving bruises.
What should you do NEXT?  

A

Ask her to bring the child in so you can verify if there are any bruises. Explain you may have to file a child
    abuse report if there are.

B

You have the option to report, but it would probably be best to not report as the client may leave therapy
C

You have no option in this case as it clearly describes physical abuse. You are required to report. Your best bet is to involve the client in this process and hope you can keep her engaged in treatment.
D

Given you did not have a chance to inform her about the legal limits of confidentially, you would work with her on her anger and parenting skills. You would not have to report


43. A 66-year-old African-American woman who sought help because she felt sluggish and sad has begun overusing her clinic visits. She recently retired from a job that she held for 40 years. She goes to a senior citizen center where she eats a hot meal every day and has told her social worker that she is still fairly active at her church, but less so than before. The woman says she feels “at loose ends” and as though her church work has less meaning than it did before. What would the social worker suspect MOST?
A

social role dysfunction due to role change
B

the onset of dementia
C

V-code religious or spiritual problem
D

depression and malnutrition


44. In tasked-centered therapy, the social worker develops a contract in conjunction with the client through

A

Goal-setting
B

Problem-solving
C

Mutual acceptance
D

Multi-tasking




45. A social worker has been working with a 35-year-old divorced woman, Joan, who has joint custody of her son, but is the custodial parent. Her ex-husband, whom the social worker has never met, is taking Joan to court in an attempt to gain full custody of the son. The social worker receives a subpoena from the husband’s attorney requesting records regarding the wife’s therapy sessions with the social worker. The social worker is required to do which of the following:

A  The social worker must release the entire record of the therapy sessions to the attorney
B   The social worker is allowed to release only those records that make direct reference to the wife’s relationship with the son, not her treatment issues
C   The social worker is required to respond to the subpoena, but is not required to release records since the subpoena was issued by the attorney and not a judge
D  The social worker does not have to respond to the subpoena because the client’s records are confidential


46. An 8-month old child displays an intense fear reaction whenever she is approached by strangers. The child's behavior can be best explained by which of the following?
 
A

The child's previous negative experiences with strangers have conditioned her to respond to strangers with fear. 
B

The child has not yet developed a schema for her mother's face and the alternate models are confusing. 
C

The face of a stranger represents an unfamiliar event that naturally elicits a fear response. 
D

The child's behavior is an abnormal reaction that is related an overattachment to her mother

47, A man who immigrated to the U.S. from Guatemala a year ago tells his social worker that he recently found a job that will allow him to make ends meet. He mentions that he is also happy because he can now send money to his parents in Guatemala. The social worker responds to this news by asking the client, “How much of your money do you plan to send them?” The social worker’s response illustrates which of the following?
A

confrontation
B

active listening
C

cultural misunderstanding
D

a useful strategy for helping the client manage his finances

48. Continued substance use despite knowledge of having persistent or recurrent problems likely to have been caused or exacerbated by use of the substance is characteristic of
 
A

substance dependence. 
B

substance abuse. 
C

either substance dependence or abuse. 
D

neither substance dependence or abuse. 



49. social worker’s new client is a single mother with two children, ages 11 and 9. The client divorced her husband five years ago, and he now lives in another state and doesn’t have much contact with the kids. The client is distressed: She was diagnosed with cervical cancer two weeks ago and is feeling overwhelmed. She is distraught about her illness and concerned about her children. The social worker should FIRST:

A refer the client to a support group
B provide supportive therapy
C give the client medical information about cancer
D help the client make provisions for her children



50. According to the “Boston Model”, the developmental course of groups can be summarized in which of the following five stages?

A  Preaffiliation (approach-avoidance), Power and Control, Intimacy, Differentiation, and Separation.
B  Meet and Greet, Exploration, Intimacy, Cohesion, Evaluation
C  Pre-Affilication, Exploration, Differentiation, Recapitulatization, Separation
D  Meet and Greet, Power and Control, Intimacy, Cohesion, Separation


51. When treating an incestuous family system, which of the following would be your lowest priority?
 
A

individual therapy for the victim 
B

individual therapy for the perpetrator 
C

aversion therapy for the perpetrator 
D

couples therapy for the parents

52. A new client has been abused by her husband, but denies that this is a problem for her. She wants help because she is having difficulty sleeping at night. In terms of addressing this client's denial, experts would recommend that you
 
A

confront her with the reality of her situation. 
B

refer her for a neuropsychological evaluation because she may be mentally confused. 
C

assuming that she is not in immediate danger, emphasize building a trusting relationship with her. 
D

assuming that she is not in immediate danger, refer her to a support group. 

53. A client has failed to pay for five sessions of therapy, even though she can clearly afford to do so. The social worker has discussed this problem with the client and attempted to resolve it in a mutually acceptable way, but the client is still refusing to pay for the sessions. Under these circumstances, the social worker could terminate treatment with this client:
 


 A only if the client has met the goals of her treatment or initiates the termination herself, because termination due to nonpayment is unethical
 B if he clearly explained his policies with regard to nonpayment at the beginning of treatment
 C  if the client is not currently a danger to herself or others and he has discussed with her the possible consequences of not paying her fee and termination of therapy
 D if the client is not currently a danger to herself or others, he explained his policies with regard to nonpayment at the beginning of treatment, and she agrees to accept a referral to another provider









54. A woman is told that she has a serious illness for which there is no cure and that it is likely that she will die within six months. From the perspective of Kubler-Ross's stages that dying people pass through, you would expect the woman's first reaction to finding out about her prognosis to be
 
A

"Why me?" 
B

"The test results must be inaccurate." 
C

"I must be being punished for something I did." 
D

"I hate my life anyway, so what difference does it make?" 



55. A social worker is providing therapy to a client through the client’s company’s Employee Assistance Program. The social worker can inform the client’s supervisor at work that the client is receiving treatment:
A

if the supervisor referred the client to the program
B

if the client’s problem is affecting her job performance
C

if that is the only information he reveals to the supervisor
D

under no circumstances at all

56. A neuroleptic would be LEAST useful for treating which of the following?
 
A

Acute mania. 
B

Tourette's disorder. 
C

Schizophrenia. 
D

Chronic aggression. 

57.  Mr. J, is referred to the Social Worker after he is released from his first psychiatric
hospitalization. His hospital admission occurred sometime after his return from a trip to Rome, where he had gone to give advice to the Pope. His family knew nothing of the trip until the bills arrived. A salesperson, Mr J. had called strangers in the early morning hours seeking sales. He was described as euphoric in mood, but his family reported that be became irrational when his wishes were thwarted. Mr. J. experienced a reduced need for sleep and was described as talking non-stop for long period of time. Based on the above information, which of the following DSMIV TR diagnoses seems most probable?
        
A   Major Depressive Disorder
B   Bipolar I Disorder, Most Recent Episode Mixed
C   Bipolar I Disorder, Single Manic Episode
D   Brief Psychotic Disorder














58. A young child has learned that objects that fly in the sky are called "birds." The first time she notices a plane in the sky, she points to it and says "bird." From a Piagetian perspective, this child's response illustrates
 
A

equilibration. 
B

adaptation. 
C

accommodation. 
D

assimilation. 



59. You are referred a client who is hearing impaired. What is the best way for you to communicate with her?
 
A

in writing 
B

hire a translator 
C

find a friend or family member who can translate 
D

study American Sign Language 


60. A lesbian couple begins treatment with you to deal with what they describe as "communication problems." They insist that they do not want to talk about their sexuality. You should
 
A

avoid dealing with their sexuality. 
B

explain that you need to discuss their sexuality. 
C

evaluate the issues they have. 
D

deal with communication problems only. 


61. You are a white-Anglo social worker who is treating an Hispanic family. The family has been in the United States for about two years. The family often arrives late for their sessions, and this annoys you. What do you do?
 
A

be aware that time may be perceived differently in their culture 
B

recognize that being late is probably an expression of resistance to disclosing family issues, as Hispanic individuals are generally uncomfortable with professional therapy 
C

consider referring the family to an Hispanic social worker 
D

invite them to discuss with you what being on time and being late means to them 


62. You are a hospital social worker seeing an Asian man, age 50, and his hospitalized elderly mother. The mother is incontinent and her memory is impaired. Her physician has suggested that she be placed in a nursing home. When you discuss this with the clients, the man seems attentive. He then states that his mother will be fine living at his home. How would you assess this situation?
 
A

the man doesn't want to discuss this issue with his mother present 
B

the man disagrees with the doctor and is convinced that he can care for his mother on his own 
C

the man is in denial about the seriousness of his mother's condition 
D

the man beleives that his mother's problem is an emotional one, rather than a phyical one 

63. The client is a 66-year-old retired man. He complains that he has been feeling depressed, isolated, and useless. What is your intervention?
 
A

refer to a support group 
B

explore activities that will give meaning to his life 
C

schedule a family session 
D

refer for vocational training 


64. At the beginning of a play therapy session, the child becomes unruly. Your best intervention is to
 
A

allow her to explore the room some more. 
B

monitor what toys she chooses. 
C

tell her you will notify her parents if she continues to misbehave. 
D

provide structure and set limits. 


65. A strategic family therapist is working with a family in which the husband and wife have avoided their own conflicts by focusing their attention on the problems of their 9-year-old son. The therapist in this situation is most likely to
 
A

educate the parents about "triangulation." 
B

obtain a detailed family history from the husband and wife to identify the pattern of interactions in their families of origins. 
C

give the husband and wife a homework assignment designed to foster recognition of the conflicts that exist between them. 
D

have the family members adopt the roles of each other and role-play a typical family interaction. 


66. A social worker’s client is Jeremy, a man with a successful business, a happy marriage, and a 7-year-old daughter. Jeremy and the social worker have been working for six months on resolving Jeremy’s feelings toward his father, who treats him like a failure. In one session, Jeremy discloses that he had a distressing dream the night before, in which he molested his daughter. He is visibly upset by the dream and confused about what it means. The social worker’s BEST action is to:
 
A

gather more information to assess if there is enough information for a child abuse report 
B

be reassuring and supportive and help Jeremy understand his feelings 
C

file a child abuse report immediately
D

take a client-centered approach and review the limits of confidentiality


67. A couple you have been seeing in marital counseling for six months decide to divorce. The husband calls you and asks that you provide testimony in their divorce proceedings on his behalf. What should you tell him?
A. You are not allowed to testify as you have never been approved by the court as an expert
    witness.
B. You can testify, but can say nothing as it relates to his wife unless she agrees to waive her
    privilege as well
C. You would be happy to testify.
D. You cannot testify legally and ethically. His waiver of privilege does not affect the wife’s right to claim privilege, even if it concerns the same information. Your client in this case was the couple, not either individual.


68. In strategic family therapy and other family therapies derived from the communication model, the primary goal of the technique known as "prescribing the symptom" can be said to be which of the following?
 
A

Undermining the family's resistance to change. 
B

Changing the meaning of a situation by changing the way it is perceived. 
C

Changing the meaning of a symptom by relabeling it. 
D

Increasing differentiation among members of the family


69.In the first meeting with a social worker, a mother reports that she is worried because a teacher found her 7 year old son touching another student “in a very sexual way” during recess. Of the following, what would be the BEST way to assess this situation?
 
A

Ask the son questions designed to uncover whether he has been abused. 
B

Through play therapy
C

Confront the mother about suspected child abuse
D

Ask the mother a series of focused, closed-ended questions to rapidly assess for child abuse 




70.. Which is the MOST commonly reported form of elder abuse?

A

Verbal abuse
B

Emotional abuse
C

Physical abuse
D

Neglect


71.A social worker asks her client, “If you were to wake up tomorrow to find that a miracle had occurred and that things were better, what would be the signs that a miracle had indeed occurred?” From which theoretical model is this social worker likely operating?

A

Psychoanalytic
B

Humanistic
C

Task-Oriented
D

Solution-Focused


72. In the initial session, a client describes the following problems: He is unemployed, his wife has threatened to leave him, and his young son is difficult to manage. His social worker should
 
A

identify the immediate problem and recommend a way to resolve it. 
B

recommend marital therapy. 
C

help the client set priorities. 
D

recommend family therapy. 




73.According to P.L. 94-142, final decisions regarding mainstreaming students into regular classrooms are made by:

A

teachers
B

local school committees
C

parents
D

school counselors


74. A new client reports experiencing periods of intense anxiety with a sudden onset. Which of the following must be ruled out FIRST?
A

hyperthyroidism
B

conversion disorder
C

diabetes mellitus
D

sexual abuse


75. During a session attended by a mother, father, and two children, the father describes an ongoing problem in the family. In order to help the family members "re-author" their experiences, the social worker would ask which of the following?
 
A

Tell me more about this. 
B

When did this happen? 
C

What happened next? 
D

What is your involvement in the problem? 


76. It is MOST recommended that social workers and other therapists minimize direct eye contact when working with:.
 
A

Caucasian clients 
B

African-American clients  
C

American Indian clients  
D

Asian clients  



77. The underlying premise of Albert Ellis's Rational Emotive Therapy (RET) is that dysfunctional behaviors are
 
A

the result of irrational thoughts and beliefs. 
B

the result of incongruence between self and experience. 
C

the result of "automatic thoughts." 
D

the result of a lack of "awareness." 



78. In an enmeshed family in which an overcontrolling father constantly nags and yells at his 18- and 15-year-old sons, a structural family therapist is most likely to do which of the following?
 
A

Manipulate the family's mood by nagging even more and yelling even louder. 
B

Help the father understand this his nagging and yelling elicit oppositional behaviors from the boys. 
C

Construct a genogram of the father's family. 
D

Create a therapeutic double-bind by telling the father to set aside an hour a day during which he will nag and yell at his sons. 



79. During the course of group therapy, some members begin to express anger toward a female member because they feel she is "hiding behind a facade" rather than revealing her true self. Other members say she should be left alone. According to Irving Yalom
 
A

conflict represents a normal stage in the group process and should be allowed to run its course. 
B

conflict is a manifestation of resistance that should be interpreted as such by the therapist. 
C

conflict should be resolved in a democratic manner. 
D

conflict should be expressed and then processed to clarify its meaning to members. 


80. Your client, who was just raped, knows who attacked her but does not want to press charges. She lives in a small town and her husband has a high-profile position with the town's largest corporation. What is your first responsibility?
 
A

to call the police 
B

to explore whether the client has received medical attention 
C

to gather information about the attack 
D

to meet with her husband and remind him of their responsibility to the community 


81. A low-income male client needs urgent home placement. Among the following, the social worker’s BEST action is to:
A

refer him to a friend of hers who rents inexpensive apartments
B

rent him an apartment that she owns
C

refer him to an apartment that she used to own
D

refer him to an inexpensive apartment with no access to public transportation


82. As a social worker in a child abuse center, you create a group for abusive parents. During the first session, one of the parents becomes angry and starts to argue with another parent. You should handle this situation by
A

Asking the parent to explain what caused the anger
B

Having the parents work through the disagreement in an appropriate manner
C

Insisting the parent stop arguing
D

Allowing the parent to vent her anger



83. In an interview with a social worker, a 15 year old reveals that her stepfather molested her when she was a child. To enter this information in the process notes, the social worker would:
 
A

record it word for word
B

summarize it and include subjective impressions
C

include only objective information 
D

Interpret it



84. "Games" and "scripts" are of interest in
 
A

Gestalt therapy. 
B

Rational-Emotive therapy. 
C

Transactional Analysis. 
D

Reality therapy. 


85. For a family systems (Bowenian) therapist, when a family exhibits a high degree of fusion but one member is more differentiated than the others, the likely approach would be to
 
A

work with the least differentiated family members. 
B

work with the most differentiated family member. 
C

work with all family members as a group. 
D

work with all family members in individual therapy until they reach a similar level of differentiation


86. You are working as a consultant for a school district. The principal of a junior high school is refusing to put a student in a special education class because the teacher involved says the student is just lazy. Your best course of action would be to
 
A

have the student evaluated to determine whether or not he should be placed in a special education class. 
B

speak to the teacher and make a referral for therapy since his personal issues are interfering with his work with the child. 
C

use previous test results on the child and draft a formal letter to the principal outlining your recommendations. 
D

obtain consent from the child's parents before confronting the principal and teacher about their unprofessional behavior. 


87. Which of the following defense mechanisms is inaccurately illustrated?
 
A

Reaction formation: A woman who hates her sister always buys her sister expensive gifts. 
B

Displacement: An unfaithful husband accuses his wife of having an affair. 
C

Sublimation: A man with high levels of aggressiveness becomes a surgeon. 
D

Regression: A college student acts very child-like whenever she goes home for vacations. 



88. Cognitive therapy would be contraindicated for
 
A

depression. 
B

an eating disorder. 
C

PTSD. 
D

delusions. 


89. Alan is an 8-year-old boy who often wakes up in the morning complaining that he has wet the bed. His parents report that this happens approximately three times per week. When asked how long Alan has been wetting the bed with this frequency, his parents stated that it has been going on for a few months. All of the following are true EXCEPT:
A

Alan likely has an Elimination Disorder
B

Alan likely has Enuresis
C

Alan likely has Encopresis
D

The therapist should continue to assess the situation to determine if there have been any major life changes during this time period or the presence of medication


90. You have just started seeing a couple in conjoint therapy and believe that the husband is physically abusing his wife. Most likely you will
 
A

wait for the husband or wife to bring up the issue of abuse. 
B

say that you want to see the wife in individual therapy. 
C

ask the couple how they deal with conflict and disagreement. 
D

ask the couple to sign a "no-violence" contract


91. As part of an evaluation of a third-grade girl referred for her disruptive classroom behavior, the school social worker interviewed the girl's father. The father related that, whenever the girl acted out at home, the parents would make every attempt to be understanding and accepting and would try to reason with her. The father reported that this usually worked in getting the daughter to stop. From a behavioral view, the social worker would reason that
 
A

the girl probably would be better off in a smaller class with less stimulation from competing sources. 
B

parental attention might be reinforcing the girl's disruptive behaviors. 
C

the parents' behavior was negatively reinforcing the girl. 
D

the girl probably modeled her behavior after someone in the family. 

92. The primary difference between reinforcement and punishment can be best described as a difference between
 
A

stimuli and responses. 
B

pleasant and unpleasant stimuli. 
C

presenting and removing a stimulus. 
D

increasing and decreasing a behavior. 


93. A child, age 4, is brought to the community mental health clinic by her Foster Care Social
Worker. The Foster Care Worker describes her former situation as being characterized by persistent disregard of the child's emotional needs for comfort, stimulation and affection. Her current behavior demonstrates indiscriminate sociability with marked inability to exhibit appropriate selective relationships. She is excessively familiar with relative strangers. Based on the above information which of the following DSMIV TR diagnoses seems most appropriate?

A   Separation Anxiety Disorder
B   Reaction Attachment Disorder, Inhibited typ
C   Reaction Attachment Disorder, Disinhabited type
D  Attention-Deficit/Hyperactivity Disorder













94. You are about to terminate a successful therapeutic relationship with a client. As you begin the discharge phase, the client regresses to some of the symptoms she exhibited at the beginning of treatment. From a psychodynamic perspective, how would you describe what is happening?
 
A

you have overlooked important symptoms 
B

the client has not developed sufficient insight 
C

the client is relying on defense mechanisms 
D

you have mishandled the termination phase 


95. A social worker’s new client is a man whose wife was recently raped. The wife is in individual therapy with another therapist. The man tells the social worker that he feels upset and confused. He says he wants to support his wife but feels distant from her, both emotionally and sexually. The social worker’s BEST approach to treatment is to:
A

normalize the client’s feelings and provide support
B

refer the client to a group for men whose partners have been assaulted
C

call the wife’s therapist to consult and coordinate treatment
D

bring the wife into couples therapy with her husband


96. A Iraq War veteran complains that since coming home from the war, he has lost the use of his left hand. His physician could not find a medical basis to explain this problem. The man says that during the war, he fired his gun with his left hand and killed several people. According to DSM-IV-TR criteria, the MOST likely diagnosis for this client is:
 
A

malingering
B

conversion disorder
C

posttraumatic stress disorder
D

 factitious disorder


97. An advocate of client-centered therapy would most likely view diagnosis as
 
A

the starting point of therapy. 
B

an ongoing process. 
C

a joint task of the therapist and client. 
D

unnecessary and possibly detrimental. 

98. The parents of a teenage boy come in just after finding out that the boy is gay. They ask the social worker to help them arrange to place their son in a psychiatric facility. The social worker’s BEST intervention is to:
A

provide relevant education and literature and explain that hospitalization is not possible in this situation
B

provide relevant education and explore the parents’ feelings
C

provide support and education, including explaining that this could be an experimental phase that their son will outgrow
D provide support and referral to a support group for parents of gays and lesbians



99. A social worker has been working for five months with a client who is clearly depressed (his diagnosis is major depressive disorder). The client has been depressed for nearly a year, but is not a danger to himself. The client has shown no improvement in his depressive symptoms since beginning to see the social worker. The social worker has repeatedly recommended that the client see a psychiatrist for a medication evaluation, but the client has refused each time. In this situation, the social worker’s MOST appropriate action is to:
A discuss the possibility of termination and referral, assuming that the client continues to refuse to see a psychiatrist
B refer the client for inpatient psychiatric observation and treatment
C see the client more frequently for psychotherapy
D respect the client’s right to self-determination and proceed with therapy




100. In treating a client whose problems seem to be due primarily to low self-esteem, a cognitive therapist is most likely to emphasize
 
A

convincing the client, through verbal persuasion, that he actually has many talents and abilities. 
B

helping the client identify the ways in which he sabotages his feelings of self-efficacy. 
C

identifying the client's self-defeating beliefs and replacing them with more self-affirming ones. 
D

educating the client about the ways in which low self-esteem is self-reinforcing. 



101. A family begins therapy with a family therapist. The therapist relies primarily on Minuchin's structural approach to family therapy and can, therefore, be expected to

A

"join" the family system but refuse to accommodate to its affective style. 
B

use interventions that stress the family system so as to unbalance the family's homeostasis. 
C

discourage confrontations between family members in order to reduce tension and help members think more rationally about their problems. 
D

help the family construct a multigenerational genogram during the first therapy sessions to facilitate history-taking. 


102. A social worker’s new clients are a mother, father, and 7-year-old son. The family has been referred to the social worker by their physician because they were reluctant to accept medical treatment for their son. The parents confirm that their son has a congenital heart condition that is growing worse. A few days ago, the family was told that the boy needs heart surgery immediately or he will die very soon. The parents tell the social worker that they decided this morning to go against the doctor’s recommendation for surgery because it is inconsistent with their religious beliefs. They believe that their son will be healed without medical intervention and that invasive medical procedures are unholy and would, therefore, endanger not only their son but also the whole family. The social worker’s PRIORITY is to
 
A

let the doctor know what they’ve decided
B

file a child abuse report  
C

attempt to establish a working alliance with the family so that she can more effectively encourage them to accept the medical treatment
D

respect the clients’ religious beliefs and provide support



103. For a gestalt therapist, a primary goal of treatment is to help the client
 
A

integrate the present with his or her past and future. 
B

integrate the functioning of his or her mind and body. 
C

develop a "success identity." 
D

incorporate the external into the internal. 

104. Which of the following is LEAST consistent with Minuchin's theory regarding family boundaries?
 
A

In an enmeshed family, parents allow a "school phobic" child to remain at home. 
B

In a disengaged family, parents usually do not notice when their children need support or guidance. 
C

An infant in an enmeshed family receives lots of love, attention, and stimulation. 
D

Conflict is a substitute for love and affection in a disengaged family. 


105. According to Irving Yalom, a leading authority on group therapy, which of the following individuals is the poorest candidate for a heterogeneous outpatient therapy group?
 
A

A person suffering from anhedonia. 
B

A workaholic. 
C

A sociopath. 
D

A shy person. 


106. An adult client whom you have seen for several sessions begins to express suicidal ideation. What is your intervention?
 
A

assess whether the environment is safe 
B

defuse anger 
C

call the client's parents 
D

mobilize the support system 


107. Which of the following illustrates the use of negative reinforcement?
 
A

A child's usual allowance is reduced by a specific amount each time he misbehaves. 
B

A child is allowed to watch television for one hour when he finishes his homework. 
C

Restrictions on a child's activities are removed each time he helps with household chores. 
D

Restrictions on a child's activities are imposed each time he argues with his sister. 






108. A social worker is working in individual therapy with an adolescent who has been living in a foster home for about 10 years. The client says he wants to learn more about his biological family’s culture. The social worker should FIRST:
 
A

 contact the client’s biological family for information
B

 explore why the client is interested
C

 provide information about the client’s culture
D

 discuss this with the client’s foster parents

109. A client who was ordered into therapy by the court after being arrested for drunk driving insists that he does not abuse alcohol. Why might a social worker use a genogram in assessing this case?

A to assess the effects of alcohol use on the client’s long-term memory
B to estimate the potential for alcohol addiction by identifying any family history of substance abuse
C to find out whether there is a long-term pattern of antisocial conduct
D to assess the effects of chronic alcohol use on the client’s interpersonal functioning













110. A man is convinced that his wife is having an affair with a co-worker. He is unable to give you any evidence to support his belief. He then admits that he has been having sexual thoughts about a woman in their neighborhood. How would you interpret this from a psychodynamic perspective?
 
A

he is projecting his unwanted impulses onto his wife 
B

his belief stems from his own sexual desire for his neighbor 
C

he has an unconscious desire to sleep with his neighbor 
D

he is expressing Oedipal feelings toward his neighbor 


111. You are conducting a therapy group and you are now in the middle stage. You ask group members to shift from answering direct questions to talking freely about themselves. The group members show resistance to this. You should
 
A

allow them to express their resistances. 
B

explain to them the developmental process of the group. 
C

tell them that resistance is not allowed. 
D

go back to asking direct questions. 


112. A depressed client with a detailed suicide plan who has voluntarily admitted himself to a hospital leaves the facility after one day. What is his social worker's responsibility?
 
A

to initiate involuntary hospitalization 
B

to educate the client about the potential consequences of leaving the facility 
C

to educate the client about the effects of depression 
D

to schedule a follow-up appointment with the client 


113. You have seen a young woman, Martha, in your clinic three times for individual treatment of her depression. She indicated that her depression is a very long- standing problem. In addition to depression she had many pressing concerns, such as financial problems and having an affair with her married boss, Mr. Jones. You receive a call from Mr. Jones’s wife stating that your client gave her your number and further that Martha was threatening suicide. Mrs. Jones asks you if you know where Martha is, and if you know of anything going on “between Martha and my husband.” While this sounds like a soap opera, it could well be a very real situation. Think through the legal and ethical issues.

A

Immediately call your attorney.
B

Say nothing to Mrs. Jones, and immediately try to reach Martha  to see if she is indeed suicidal.
C

Say nothing and completely ignore the call. She is not your client and you cannot say anything legally to her
    about Martha.
D

. Offer to see Mrs. Jones in counseling since she obviously has issues to discuss. Immediately call the police.




114. A client complains of feeling depressed and anxious. The social worker evaluates the client for suicidal risk, evaluates her psychological status using a mental status exam, and gathers information about her social relations, cultural background, and environmental connections. What should the social worker do NEXT?
A

formulate a diagnosis based on the client’s self-report and the results of the mental status exam
B

explore the client’s health history, family health history, health and lifestyle behaviors, and medication use
C

seek permission to speak to the client’s family members and other collaterals to verify and supplement the client’s self-report
D

share the results of the assessment with the client


115. The drug Depokate is often used to:

 A  treat neurosis.
 B  aid in sleeping.
 C   treat mania.
 
D  treat learning disorders.





116. Aggressive and antisocial behaviors associated with conduct disorder are BEST treated by:

A behavior therapy and pharmacotherapy
B behavior therapy and parent training
C cognitive-behavioral therapy
D family therapy and pharmacotherapy


117. After working with a female child at school for two weeks, a social worker visits the child’s home and notices that her mother has many bruises and a black eye. Based on this and certain behavioral cues, the social worker comes to suspect strongly that the woman’s husband beats her. However, the woman clearly is reluctant to talk about how she was injured. The BEST way for the social worker to get the mother to face this problem would be to:
 
A

call a shelter from the woman’s home
B

continue to work with the woman’s child 
C

press the woman to disclose the origin of her injuries
D

confront the woman with his suspicions and offer to help in concrete ways


118. An American Indian couple with two children lives in poverty and needs help accessing resources and services. In her first meeting with these clients, the social worker learns that the husband drinks alcohol daily, sometimes to the point of intoxication. She attempts to discuss this issue with the couple, but they don’t respond to her efforts. The social worker assists the clients to meet their basic needs for adequate food and medical care for their children. Which of the following is MOST likely true about this situation?

A

the social worker should have continued her efforts to address the man’s alcohol use during the session, rather than focusing on the clients’ other needs
B

the clients don’t view the man’s alcohol use as a problem and just want help meeting their basic needs
C

the clients are ashamed of the man’s alcohol use and unwilling to talk about it with anyone outside the family
D

the social worker should have focused on only the client’s basic needs in the first session, because the clients will be willing to address the man’s problem with alcohol use after their basic needs are met



119. Two adult children report that their father is becoming somewhat confused. Neuropsychiatric tests for dementia have been negative and the father is ambulatory. The children want to place their father in an independent living facility. How do you intervene?
 
A

give the clients literature on independent living facilities 
B

suggest that the children try to assume guardianship of their father 
C

assess family dynamics 
D

do an in-home assessment of the father 


120. In a closed therapy group:
A

the group leader is more active than the group members
B

members leave the group as soon as they have met their goals
C

duration is predetermined
D

new members may join only when a member leaves


121. A 15-year-old client who you have been seeing for several months says he's been thinking about killing himself. In response to your concern, he says he's "just kidding" and asks that you not tell his parents. Your best course of action would be to
 
A

tell the boy that you need to contact his parents and then do so. 
B

tell the boy that you won't tell his parents as long as he signs a "no suicide" contract. 
C

continue to discuss the matter to determine if the boy is, in fact, at risk for suicide. 
D

do nothing since he was "just kidding" but ask him about the issue during the next session. 



122. A client you have been seeing for eight months says he wants to quit therapy. He feels that the original problems he came to therapy for have all been resolved. You disagree and feel that there is good reason for the man to continue seeing you. You should
 
A

get the client to agree to a few more sessions. 
B

discuss his reasons for wanting to terminate. 
C

discuss his reasons for wanting to terminate and your reasons for thinking he should continue. 
D

let him terminate but let him know he can come back if he desires. 



123. A single mother with three young children is unemployed and has no family members or other support people nearby. She says she would like to build her support network. What services do you provide?
 
A

respite services 
B

career counseling or vocational training 
C

support group for single parents 
D

parenting classes 


124. A 14 year old tells her social worker that she been chatting over the Internet with someone who contacted her because he liked the photo of her he saw on her personal Web page. The client says that it’s exciting to have an older guy as a friend, but that she feels “sort of weird” when he writes about meeting her or calls her “sexy.” The man sent her a photo of himself, which the client has printed out. When she shows the photo to the social worker, she says, “I think he’s kind of cute, but I don’t know ....” The man looks to be about age 30. The social worker’s BEST action is to:
 
A

tell the girl that he is going to call her parents to encourage them to monitor her use of the Internet more closely  
B

help the girl understand what is dangerous about this situation and discuss how to be safe when using the Internet
C

ask the client why she thinks it’s “exciting” to have an older man as a friend
D

let the girl know that her contact with this man is inappropriate and then talk to her parents about contacting the police



125. If you were to use a problem-solving model to treat a client, you would emphasize
 
A

motivation, capacity, and opportunity. 
B

changing his environment. 
C

modifying the person-in-situation system. 
D

helping him achieve his potential. 



126. A clinical social worker who has been seeing a client for two months notices that the client seems even more depressed than she was at the beginning of therapy. The client has had depression for eighteen months. The social worker should
 
A

refer the client to a psychiatrist for an assessment. 
B

challenge the client about her lack of improvement. 
C

refer the client to a medical doctor. 
D

refer the client to a psychologist for testing. 





127. A family member who is alcoholic gets better and a previously healthy family member develops symptoms. This is an example of
 
A

complementarity. 
B

wholeness. 
C

equifinality. 
D

homeostasis


128. Which of the following is NOT considered a part of crisis intervention?
 
A

Removing a client's symptoms. 
B

Helping a client develop coping strategies. 
C

Allowing a client to select from a variety of short-term treatment alternatives. 
D

Identifying what led to the crisis. 


129. Two women, ages 77 and 78, who live together in a small apartment have begun to have difficulty performing some activities of daily living. They lack adequate support in their community. You should
 
A

refer them to a support group. 
B

recommend homemaker services. 
C

provide literature on nursing homes. 
D

refer them to a community mental health center. 



130. A man brings his wife to a busy community medical clinic because she is not feeling well. The woman dies while waiting to be seen by a physician. The social worker should
 
A

offer to counsel the man on grief and loss. 
B

refer the man to a grief counselor. 
C

explain the stages of grief and loss. 
D

find a private room where the man can express his feelings and collect himself. 




131. At the beginning of a group therapy session, a member approaches you individually with a question. What do you do?
 
A

ask him to bring it to the group 
B

answer the question 
C

tell him you'll speak with him after the meeting 
D

schedule an individual session 



132. In contrast to classical psychoanalysis, ego psychology
 
A

relies more transference in therapy. 
B

emphasizes the role of the id in personality development. 
C

conceptualizes maladaptive behavior as resulting from intrapsychic forces. 
D

focuses on the client's current problems. 



133. Children begin to understand the concept of death in the
A

concrete operational stage
B

genital stage
C

phallic stage
D

preoperational stage

134. In which of the following situations would an exception to the requirement for an informed consent to treatment NOT be an issue?
 
A

Medical or psychological emergency. 
B

Incompetency. 
C

Client waiver. 
D

Court-ordered treatment. 



135. In child welfare work, the PRIMARY goal of permanency planning is to:

A

improve long-term continuity in the care of dependent children 
B

improve the quality of foster care placements 
C

help the biological parents of dependent children become capable of caring for their children
D

facilitate the adoption process for adoptive parents



136. Your clients bring in their young son who has a brain tumor. The day before, a neurologist told them that the boy needs to be operated on immediately, or he will die. The parents tell you that they have decided to cancel the operation based on their religious beliefs, insisting that the boy will be healed without medical intervention and that any medical intervention would actually endanger the family. What do you do?
 
A

provide relevant education 
B

file a report with a child protective services agency 
C

establish a working relationship with the family 
D

respect their preferences 



137. A client you have seen several times tells you that he had sexual intercourse with his 13-year old niece three times during the last year. You should
 
A

maintain the client's confidentiality if you determine he is unlikely to sexually molest the child again. 
B

immediately make a report to the child protective agency whether or not you think it is likely he will sexually molest the child again. 
C

immediately alert the parents of the child and make a report to the appropriate agency if they seem unable to protect the child. 
D

immediately alert the parents of the child. 



138. You receive a subpoena duces tecum from the court requesting that you provide it with information about one of your clients. Your best course of action would be to
 
A

immediately provide the information requested to the court. 
B

obtain the client's consent and then provide the information requested to the court. 
C

assert the privilege not to reveal confidential information and then provide the information requested to the court. 
D

assert the privilege not to reveal confidential information and provide the information only after obtaining the client's consent or being ordered to do so by the court. 


139. Tim Upright, a social worker, is seeing a 13-year-old girl in therapy. The girl's school guidance counselor calls, stating that the girl is having "disciplinary problems" at school and that he would like to discuss the situation with Tim to obtain some advice on how to handle the girl's problems. As an ethical social worker, Tim should
 
A

provide the counselor with whatever information he needs. 
B

provide the counselor only with information relevant to the girl's problems at school. 
C

provide the counselor with information only after obtaining written permission from the girl's parents. 
D

refuse to provide the counselor with information since he is not a licensed social worker. 


140. By the third session, Karen Novice, a social worker, realizes that a client would benefit most from a type of intervention with which Karen has no previous experience. As an ethical social worker, Karen should
 
A

continue seeing the client and use treatment methods with which she is familiar. 
B

continue seeing the client but read the literature on the intervention before using it to treat the client. 
C

refer the client to another therapist whom she knows is experienced with the intervention. 
D

advise the client of her limitations and let the client decide whether or not to continue therapy with her. 






141. Most states in the nation would require a child abuse report to be filed in which of the following situations?
 
A

a 12-year-old and a 13-year-old are touching 
B

a 13-year-old and an 18-year-old are touching 
C

a 16-year-old and an 18-year-old are having intercourse 
D

a 16-year-old and a 20-year-old are having intercourse 




142. You've been seeing a client for eight months. During that time, he has complained several times about the course of therapy and, each time, you have discussed this matter with him and modified treatment goals. During the last three sessions, he has again mentioned that he is not happy with therapy. As an ethical social worker, your best course of action would be to
 
A

realize that his behavior is a manifestation of resistance and ignore it. 
B

make a clinical decision as to whether or not the client is benefitting from therapy and, if you determine that he is, encourage him to continue in therapy. 
C

discuss his dissatisfaction with him and the possibility of a referral to another therapist. 
D

tell him that, as an ethical therapist, you must terminate treatment since he does not feel he is benefitting. 



143. A child in the preoperational stage of development is told that his parents have gone out of town for a few days. His reaction to this information is MOST likely to be which of the following?
A

he will believe that his parents are gone forever
B

he will ask, “How long is a few days?”
C

he will ask why they went out of town
D

he will search for his parents


144. You are dining at a restaurant with a colleague when one of your clients stops by your table to say hello. How do you respond?
 
A

remind the client about confidentiality 
B

introduce the person as your client since you are dining with a colleague 
C

acknowledge the greeting in a cordial way and tell the client you will call her later 
D

tell the client you will discuss this event in an upcoming session 

145. You provide outreach services in the aftermath of a natural disaster in your community. The next week, you decide that you would like to publicize the good work you did. What do you do?
 
A

contact the individuals you assisted and solicit testimonials from them 
B

contact the mayor and ask him or her to issue a public statement about your work 
C

call the local media and inform them of your work 
D

nothing; this would be unethical 


146. A worker describes the client as "just sitting there while I do the work. I hope he cancels the  appointment. I am thinking about referring the client somewhere else." These feeling on the worker's part should alert the worker to the possibility that:
        
 A  goals are not mutual.
 B   the client cannot be helped.
 C  the worker client relationship is hopeless.
 
a referral is the best plan.













147 Hospice care is designed for:
A

people recovering from physical illness whose physicians prescribe it
B

people who cannot afford to pay for hospital care and have no health insurance
C

people at the end stage of terminal illness
D

people at the end stage of terminal illness who have no one to care for them  


148 A teenager with a history of aggressive behavior is referred to a social worker from a juvenile detention facility. The client used to initiate physical fights with other students at his school. He also has a history of school truancy, theft, and alcohol use. In an early session with the social worker, he says, “This kid I know, Jack, is a real pain; I’m going to get him.” The social worker should:
A

maintain the client’s confidentiality
B

inform the police and have the client sent back to the detention facility
C

inform the client of the limits to confidentiality and her legal obligations
D

have the client sent back to the detention facility


149.  Which of the following is the MOST dangerous side-effect of the medication Prolixin?
A

blood disorder
B

seizures
C

palpitations
D

tardive dyskinesia

150. Steven is an 18-year-old college student who you have been treating for specific phobia. Over the last 8 months, he has made great strides and is feeling much more confident. In your last session, he explains that, while it has not had a major impact on his life, he has always been bothered by the fact that he will often pass out at the sight of blood. While you have treated specific phobias before successfully with systematic desensitization, you have never treated a blood-injection type phobia. You are unsure if the normal way you go about treating phobias will work because “passing out” is different from the expected phobic response of fear. When you tell Steven that you have no experience treating his condition, he says that you are the only therapist he has had that he trusts, and would not feel comfortable seeing anyone else.  The Most Appropriate action is to ?

A. Get enrolled in a course teaching you how to treat blood-injection type
    phobia and learn it as fast as you can.
B. Immediately refer Steven to a colleague who has the particular skill. You
    cannot provide services if you have not had graduate school training in this
    area
C Treat Steven anyway because you do have experience treating other
    phobias, and if you f ail to treat Steven, you would be abandoning your client.
D Seek information, consult with colleagues, or make a referral if
    appropriate.

















151.   Depression in the elderly is:

         A  well recognized by care givers.
        
B  masked by somatized complaints.
        
C  not a frequent problem.
       D  screened for by most agencies.













152. A client you have been seeing in individual counseling for 8 weeks concerning issues of domestic violence. In today’s session, he asks you if you would help him stop smoking. He desperately wants to work on this issue, and states he is not willing to continue to discuss his issues of explosive anger (which brought him into counseling) until he is successful in his attempt to stop smoking. You do not have any background or training in smoking cessation treatment. Your best response under the circumstances is:

A Given his unwillingness to address what you consider his primary issues, you agree to help
    him stop smoking, but only for a limited number of sessions.
B Offer to refer him to another therapist or program that specializes in smoking cessation while he continues his anagermanagement with you.
C Tell him you consider his desire to exclusively address his smoking as an attempt to avoid
    addressing his real issues of his anger and violence

D Tell him you would consider addressing his smoking only after he has
    shown his willingness to address his anger and violence issues.

153.  A man is reprimanded by his boss for not completing a project on time. The man is upset by this because he has never missed a deadline before, but he accepts the criticism without responding to it. That night, he yells at his wife when she enters his den while he is working on the project. He rarely yells at his wife. The man’s behavior illustrates: 
A

projection
B

displacement
C

sublimation
D

reaction formation














154.  The client, age 15, is a American Indian who was adopted by a white family when he was a young boy. His social worker begins talking about the client’s American Indian heritage, but the client expresses no interest at all in this and changes the subject. The MOST likely reason for this is:

A difficulties with his adoptive parents
B abuse in his biological home
C biculturation
D the boy’s developmental stage


155.
A client is extremely concerned about the loss of some hair. The hair loss is mild and not noticeable to others. The client refuses to leave the house without a hat on, no matter how hot it is outside. The MOST likely diagnosis for this client is:
 
A

 body dysmorphic disorder
B

 conversion disorder
C

 social phobia
D

 somatization disorder

156. During a session, a client tells her social worker something that makes the social worker realize that he and the client attend the same church. The social worker’s BEST action is to:
 
A

find a different church to attend 
B

avoid contact with the client at church 
C

 seek consultation about this
D

let this client know about this and provide an appropriate referral


157.  A gay client recently revealed to his pastor that he is gay. The client has been an active member of his church for many years and decided that he wanted the people there to know the “real” him. He has come to regret his decision because the pastor and other people at church are now avoiding and excluding him. Their behavior is making him question his faith. The social worker’s BEST intervention is to:
 
A

 explore the decision-making process that led the client to disclose his sexual orientation at church
B

validate the client’s feelings and encourage him to seek support elsewhere
C

 explore the client’s diminished religious faith
D

 refer the client to a church where he will find acceptance

158. When acting as a mediator, you should:
A

explain to both parties your opinion on the issues in question
B

attempt to get both parties to be more flexible about their goals
C

make the final decision if the parties can’t agree 
D

help the parties understand and work through their differences in order to improve their relationship with each other


159.  A woman who recently immigrated to the United States from El Salvador has been convinced by her parents to seek help for frequent alcohol and marijuana use. The social worker should FIRST:
 
A

 interview the parents
B

 explore acculturation problems
C

 take a substance use history
D

determine the appropriate level of care

160. You receive a letter from the Board of Examiners for Social Work licensure saying that one of your clients from your clinic has filed a complaint. The complaint alleges that you violated the profession’s Code of Ethics. Because a Code of Ethics is not the same as law, you should be
A

Just as concerned than if you violated the law.
B

Not concerned
C

More concerned as if you violated the law. You can lose your license, a fate worse than
    jail
D

Less concerned than if you violated the law. The worse that can happen is the licensing
    board sanctions you.



161.  A social worker is referred a client who has just discovered that she has a terminal disease. The referring physician says that the client wants help writing her will, and the client says the same thing. The social worker should:
 
A

help the client write the will
B

connect the client to a lawyer
C

explore how the client has reacted to her diagnosis
D

find out whether the client wants counseling or psychotherapy


162.  A key goal of Berne’s transactional analysis is: 
A

for the client to learn to rely on her “adult” ego state when making decisions
B

for the client to develop and follow a more adaptive “life script”
C

for the client to make new decisions that reflect, in part, an integration of all three ego states 
D

for the client to learn more adaptive “games” that promote the development of a “success identity”

163 A social worker has been providing individual therapy to an 18 year old whose parents are paying for her treatment. The client has not been getting along with her father, who treats her “like a child.” During a session, the client tells the social worker that she feels ready to confront her father about this. A few days later, the client’s father calls the social worker. He is upset and asks the social worker why she encouraged his daughter to confront him. The social worker should:
 
A

validate the father’s feelings and answer his question briefly, focusing on only the progress his daughter is making in therapy
B

validate the father’s feelings and let him know that she and his daughter will discuss this event the next time she comes in
C

encourage the father and his wife to come in for a family session
D

 tell the father that she cannot talk with him about this


164. A  client tells her social worker that she wants to join a therapy group. All of the following would suggest that the client is a NOT good candidate for group therapy, EXCEPT:
A

she has been resistant in individual therapy
B

her motivation to change is low  
C

she is severely depressed
D

 she has low self-esteem





165. According to psychoanalytic theory, a patient with an obsessive-compulsive neurosis is likely to strongly rely on which of the following sets of defenses?
 
A

reaction formation, isolation of affect, undoing
B

 reaction formation, projection, displacement
C

 somatization, introjection, undoing
D

 denial, repression, intellectualization

166.  The technique of “time-out” is MOST associated with:

A

a humanistic approach
B

solution-focused therapy
C

a social learning approach
D

Beck’s cognitive therapy


167. A social worker has been treating a 7-year-old boy who was physically abused by his stepfather in the past. In a session, the boy uses several dolls to play out a violent situation. The social worker believes that this play behavior may indicate that the boy’s feelings of anger are surfacing. To understand the boy’s feelings, the social worker should: 
A

 encourage the boy to explain his game to her
B

 ask the boy to play out another scene
C

interpret the game for the boy
D

 ask the boy why his game is so angry



168.  A client with chronic, severe back pain tells his social worker that his doctor prescribed an opioid analgesic for him, but that he doesn’t want to take it because several of his close relatives have drug abuse problems and he, himself, used to drink “too much alcohol.” The social worker’s BEST response is to:
A

support the client’s decision and explore with him alternative approaches to pain management 
B

offer to talk over this issue with the client’s doctor
C

explain to the client that using an opioid to manage pain won’t result in addiction if he uses the drug as prescribed
D

 support the client’s decision and use cognitive-behavioral interventions to treat his pain

169.  Psychoanalytic theory proposes that which of the following defense mechanisms underlies paranoia?

A

reaction formation
B

 introjection
C

projection
D

displacement  


170. An attorney phones to request a copy of one of your client’s records. Your FIRST reaction is to
 
A

 refuse to release the records
B

 release the records
C

 have the client sign a release of information
D

find out what information the attorney is seeking

 ANSWERS

1.       C  The use of candor mean the social worker will use openness and sincerity to build a trusting relationship with the teenager.  The teenager  oppose a social worker who is  authoritative and secretive. The first thing the social worker should do is start where the client is by letting the client know you are there to listen and offer support. Teenagers usually need to know that they are valued, accepted and cared for in order for them to trust you. The social worker  should not tiptoe around the issues but be open about what is going on and invite the teenager to ask questions and share their feelings. A,B,D are steps you may take later in treatment.

2.       C.Psychoanalysis  theory first founded by Sigmund Freud. It is a psychodynamic approach which views human behavior  as being motivated largerly by unconscious processes. Psychoanalysis proposes early development problems such as child maltreatment will have a profound effect on adult functioning such as parenting, social and occupational functioning.

3.       B. Cognitive Behavioral Therapy. is used with individuals who can understand that can incorporate learning theory and cognitive psychology.  A client with pervasive developmental disorder refers to a group of developmental conditions that involve delayed or impaired communication and social skills,behaviors and cognitive skills (learning)

4.       D. According to psychoanalytic theory the superego emerges when the child is five years of age which also the time when the child is able to postpone gratification of the id’s instincts. During the phallic stage, a child between the ages of 3-5, is faced with the primary task of the resolution of the Oedipal Conflict, which is marked by a desire for the opposite-sex parent as arrival. A successful outcome results from identification with the same sex parent.

5.       A.According to the NASW Code of Ethics, there are 6 core values; service, social justice, dignity and worth of the person, importance of human relationships, integrity and competence.

6.       D.This type of ethical dilemma may be especially acute for social workers in healthcare settings who are privy to medical and genetic information that may affect the welfare of others.  Social workers should always provide psycho-education about safe sex, explore client’s perception of HIV and clarify and possible biases in relations to the disease first. 

7.       B. Beck (1988) comments on the difficulty of predicting violent behavior, especially for client who are likely to be impulsive, such as those  with certain mental disorders. Until now, whevenever violence appears to be impulsive for whatever reason, courts are reluctant to find the defendant negligent” for not warning a potential victim(Beck, 1988,p.381) NASW code of ethics encourages social workers to disclose confidential information to prevent serious,foreseeable and imminent harm to a client or other identifiable person”(1999,1.07c) The limits on duty to protect have been codified in the state stautes that explicitly make reference of the Tarasoff Duty.

8.       A.  Social worker is experiencing an ethical dilemma because she was hired to provide therapy to students not to do crisis intervention.  The social worker’s most appropriate action is to explain competing values with the teacher.

9.       D. Bipolar I is characterized in the DSM as a person experiencing  or or more manic or mixed episodes. Often, but not always,there is a history of major depressive episodes. A manic episode of at least one week with symptoms of grandiosity,decrease need for sleep,pressure to keep talking,flight of ideas,distractability,increase goal directed activity, increased pleasureable activities;shopping sprees,sexual indiscretions,foolish investments)

10.   D The current school setting is not addressing the child’s needs. A comprehensive evaluations would assess client’s social, cognitive and psychological needs; which would be in the best in interest of the client.
11.   A. Anxiolytic are used to treat anxiety. It is characterized as a sedative
12.   C. Whenever client’s are experiencing stressors or losses Social Workers ought to address them in the session and provide resources to families without support in order to stabilize the family.
13.   A.  Social worker is experiencing a value conflict  regarding self determination and morals. Personal values can cause  biases and stereotypes and can have negative impact on the therapeutic relationship. It is  in the best interest of the clients that the social worker seek supervision to address countrertranference issues and if value conflict continues to impact the therapy case should be tranfered to another social worker.
14.   C. It appears that the social worker did not discuss morality with the client. Stealing is an unacceptable crime that applies to everyone in society which should be discussed at the beginning of treatment.
15.   B. Schizoid is characterized in the DSM as a  pervasive pattern of detachment from social relationships and a restricted range of emotional expression in interpersonal settings. People with this disorder do not want or enjoy close relations; they prefer to be loners.
16.   D.The diagnostic criteria for Histrionic Personality Disorder is a pervasive pattern of excessive emotionality and attention-seeking behaviors such as;constant demands for approval,inappropriate exaggeration of emotions,discomfort when not in the center of attention, inappropriate sexual seductiveness and consistent use of physical appearance.
17.   . A. It is important to start where the client is. B,C,D are steps that may be taken later in treatment.
18.   . D. D is too vague and it does not explain the reasons why cultural diversity is beneficial to clients.
19.   A.  Impulse Control disorder is the failure to resist an impulse,drive temptation to perform an act that is harmful to the person or others.  Trichotillomania is te recurrent failure to resist impulses to pull out one’s own hair. Intermittent Explosive Disorder,kleptomania,pathological gambling and pyromania are also classified as impulse control disorder.
20.    C. Pain is the focus of the clinical presentation and it cannot be fully accounted for by a general medical condition. Psychological  factors are judged to have an important role in the onset, severity, exacerbation or maintenance of pain.

21. A Beck and his colleagues have conducted several studies to identify the factors that are most associated with suicidal behavior. Not surprisingly, perhaps, they have found that feelings of pessimism and a sense of failure are the best predictors. In other words, the research has found that the cognitive characteristics associated with hopelessness are most highly correlated with suicide attempts. Consequently, a goal in the treatment of suicidal patients is to increase hopefulness.

22.B The prognosis in schizophrenia varies with the type of reaction, the particular patient, and the nature of the patient's home situation. Note that the question is asking about the type of reaction that does not have a poor prognosis. The prognosis for schizophrenia is better when the onset is acute (sudden) rather than insidious. Earlier onset (e.g., in adolescence), lack of emotional responsivity, and the presence of obsessive-compulsive symptoms are all associated with a poor prognosis.

23. A Recurrent or persistent depersonalization is an essential feature of depersonalization disorder. Even if you are unfamiliar with this disorder, its name ("depersonalization") suggests its symptoms. The DSM defines depersonalization as an "alteration in the perception or experience of the self so that one feels detached from, and as if one is an outside observer of one's mental processes or body (e.g., feeling like one is in a dream)." Thus, depersonalization can be described as feelings of disorientation and lack of ego functions. Alternative "b" is incorrect because, although anxiety might be a result of depersonalization, it is not the best description of it. Response "c" is incorrect because uncontrollable thoughts are characteristic of obsessive-compulsive disorder, not depersonalization. Alternative "d" is wrong because extreme fear and avoidance are characteristic of phobias, not depersonalization.

24. A The use of the plastic bags in this manner is associated with "autoerotic asphyxia," which refers to asphyxia caused by intentionally strangling oneself while masturbating; this behavior, which may or may not be suicidal in intent, is based on the person's belief that reduced oxygen flow to the brain can enhance orgasm. A variety of physical aids may be used, including a plastic bag, hanging platform, or a basin of water. Teens abusing inhalants ("b") may use paper (rather than plastic) bags: e.g., Glue sniffers may distribute the glue on the inside surface of a paper bag; the solvents commonly found in glue then disperse into the air inside the bag, and are inhaled or "huffed" by the abuser. Although the boys' behavior may be considered deviant ("d"), the term autoerotic asphyxia specifically describes the likely intent of their behavior.

25. A Family therapy is frequently an important adjunct to other treatments for patients with substance use disorders. This is true, first, because the addiction of one family member can have profound effects on other family members, and second, because addressing family problems greatly increases the probability that the substance addicted patient will be able to maintain his sobriety. Enlisting the support of family members is particularly important when the patient lives with his family and has a dual disorder, as in this case. Family members need to be educated about how to provide support, what to expect in terms of functioning and relapse potential, how to avoid "enabling" behavior, and the patient's medication regimen and potential medication side-effects. The patient would have been given a referral to a 12-step group ("b") from the residential facility staff. And although maximizing the patient's occupational functioning will be an important treatment goal, the priority at this time is to stabilize him after his discharge, and intervening with his family is a more relevant and effective way of doing this.

26. D Feeding disorder of infancy or early childhood (a.k.a. failure to thrive) involves a persistent failure to eat adequately, along with a significant failure to gain weight or significant loss of weight. According to the DSM, parental psychopathology, child abuse and neglect, and problems in the parent-child relationship are frequently associated with failure to thrive, and should be an assessment focus. For instance, infants with feeding disorders are often highly irritable and difficult to console during feeding, and, in some cases, parent-child interaction problems may contribute to or exacerbate the child's feeding problems (e.g., the parents may have inadequate knowledge about how to respond to the baby's needs and may respond to his or her food refusal as if it were an act of aggression or rejection).

27. C An IQ of 30 is associated with severe mental retardation; by adulthood, many (though not all) individuals with severe mental retardation are able to develop the skills listed in answer "c." The skills in answer "a" are associated with mild mental retardation (IQ 50-55 to about 70); the skills in answers "b" and "d" are associated with moderate mental retardation (IQ 35-40 to 50-55).

28. D Schizotypal personality disorder is characterized by social and interpersonal deficits and cognitive or perceptual distortions and eccentricities. Individuals with schizotypal personality disorder usually have few friends and are uncomfortable around most people. They may say they want closer relationships, but they generally prefer to stay alone. Choice "a" better describes schizoid personality disorder; response "b" sounds more like paranoid personality disorder; and alternative "c" is symptomatic of avoidant personality disorder.

29. C In the somatoform disorders (e.g., somatization disorder, conversion disorder), there are clear and existing physical symptoms that are suggestive of a somatic disorder but there is no detectable organic or neurophysiological dysfunction that can fully explain the symptoms; there is a strong presumption that the physical symptoms are linked to psychological factors.

30. B Behavioral assessment is used as both a diagnostic and evaluative tool to guide decisions related to interventions and to assess their effectiveness. Behavioral assessments focus on samples of behaviors rather than signs of underlying phenomena. The other responses accurately describe the characteristics of behavioral assessments.

31.
D
Only “d” is true. Encopresis is characterized repeated passing of feces into inappropriate places. The symptom is usually involuntary, but can be intentional (which rules out “a”); there is at least one soiling event per month for three or more months (which rules out “c”); and the child’s chronological or mental age must be at least 4 years. In addition, the disorder cannot be due to a general medical condition or the use of laxatives or other substances. Option “b” is not required for a diagnosis of encopresis. In earlier versions of the DSM, encopresis and enuresis were both coded as either “primary” or “secondary.” DSM-IV-TR no longer includes this distinction for diagnostic purposes, but does discuss it when describing the expected course of an elimination disorder: Encopresis (or enuresis) is “primary” if the person has never established fecal (or urinary) continence (the ability to control elimination); and “secondary” if the disorder develops after a period of established fecal (or urinary) continence.

32. D Although you would want to obtain information about the client's social situation, ruling out a medical disorder would be more important for determining an accurate diagnosis and treatment plan. Depending on whether a medical condition is confirmed or ruled out by the medical professional, other forms of assessment, such as a referral for psychological testing, could be appropriate.

33. A While all of these indicators could tell you something about a client's level of motivation, the most telling factor would be his reason for coming to therapy. A client who seeks therapy voluntarily ordinarily is more motivated than one who comes in involuntarily (e.g., at the insistence of a family member, because he is referred by the courts).

34.
B
You needed to choose the answer that does not describe a purpose of role-playing. Some role-playing involves having a client rehearse a behavior that will be useful in an anticipated situation because it will enable her to achieve a goal or fulfill an expectation (“a”): The client practices the behavior and receives feedback from you. When used in this way – i.e., in behavioral rehearsal – role-playing increases the client’s sense of self-efficacy. Similarly, modeling through role-play is effective for helping a client learn vicariously new behavior and reducing her anxieties about performing the behavior. When you model a behavior, you ask the client to play the role of the other person involved in the situation, which helps her anticipate that other person’s behavior. Other role-playing involves having the client re-experience something from her past by imagining that she is someone else, such as her mother or father. This form of role-playing increases the client’s self-awareness and understanding of others (“c” and “d”). By contrast, role-playing is not used to understand the cause of a client’s social role ambiguity: “Role ambiguity” occurs when the client has a lack of clarity about the role expectations associated with one of her social roles and can result in insecurity and interpersonal conflict.

35.  B. A classic description of an Histrionic Personality. The need for being the center of attention and the sexually seductive behaviors are big clues

36. C Baumrind (1972) identified three types of parenting styles: Authoritarian, authoritative, and permissive. Authoritative parents combine rational control with warmth, receptivity, and the encouragement of independence and produce children who are independent, self-assertive, self-confident, self-controlled, and content. Authoritarian parents, who impose absolute standards of conduct, stress obedience, and use force to obtain compliance, are likely to produce children like those described in response "a." Permissive parents, who provide their children with few controls or demands and display moderate levels of warmth, are most likely to produce children like those described in response "b." No parenting style in particular has been associated with the set of traits in alternative "d."

37.
D
When working with an involuntary client who has been mandated to seek therapy, a social worker should open the first meeting by telling the client the factual information she has about why he has been referred to her and required to seek professional help. After sharing this information, the social worker should invite the client to tell his side of the story. If the client alleges any contradictions in the two versions of events, the social worker should tell him that she will seek clarification. A social worker should be aware that a client who has been forced to seek professional help may have negative feelings about therapy (“a”), but the question doesn’t describe this client’s feelings. Moreover, “d” describes the most appropriate way to open the first interview with an involuntary client. If the client ends up expressing negative feelings, however, then the social worker should address them directly, using her interview skills to help the client express his feelings. Ru d set a negative tone to open the interview this way. Finally, identifying a goal that the client is motivated to le out “b”: The social worker will inform the client of the consequences of not cooperating, but it woul work on (“c”) is one effective strategy for engaging an involuntary client, but this strategy is more appropriate later on.

38.
D
You needed to consider all of the information in the stem. While “a” might be a good thing to do at some point, it doesn’t address the client’s immediate concern. Option “b” seems unnecessary because the source of the client’s confusion is already apparent. If the client is interested, this issue could be examined more closely later but emphasizing it now doesn’t address the client’s primary need in a direct enough way. In addition, the client’s concerns about having premarital sex seem to be related directly to his religious faith and training, so there is no good reason to suspect molestation. “Reassure” is OK, but the rest of answer “c” sounds judgmental. Option “d,” on the other hand, relates well to the information we have. The question tells us that most of the client’s confusion stems from his sense of conflict between his religious training and strong faith on the one hand and what he sees his peers doing on the other.


39.
B
Solution-focused therapists de-emphasize the causes of problems and emphasize helping clients become “unstuck” and focus on solutions. Stating goals in positive concrete (behavioral) terms helps the client recognize when he is accomplishing his goals and is a good predictor of outcome in solution-focused therapy.


40. D Erikson identified eight stages of psychosocial development: Oral-sensory, muscular-anal, locomotor-genital, latency, adolescence, early adulthood, middle adulthood, and maturity. Each stage involves a different psychosocial conflict. The anal stage of development occurs during the second year of life. The task of the anal stage is to achieve muscular control, which leads to the development of a sense of self-control (autonomy). Failure to gain self-control causes shame and doubt. Alternative "a" is characteristic of adolescence; choice "b" is characteristic of the latency stage; and response "c" is characteristic of the oral-sensory stage.

41. C The item draws attention to the Social Worker's responsibility to both parents and also points to issues of "best interest of the child." The outside expert who knows nothing about either parent and who is responsible to the court is the cleanest and best way.

42. C


You have no option in this case as it clearly describes physical abuse. You are required to report. Your best bet is to involve the client in this process and hope you can keep her engaged in treatment. You are legally and ethically mandated to report child and abuse and neglect

43.
A
Retirement produces the loss of a familiar and often valued social role. For any person, especially one who has been working for most of her life, retiring can lead to feelings of depression and social-role functioning problems. The client’s race may also play a role in this case: African-American individuals tend to be group-centered and work-and education-oriented and to have strong kinship bonds (McCollum, 1997). The V-code religious or spiritual problem (“c”) is tempting, since the woman says that her church work has less meaning than it did before; her symptoms, however, involve more than just this, and “a” is a more encompassing choice. Regarding “d,” although depression is possible, malnutrition is improbable because the woman is eating at least one hot meal each day at the senior citizen center.


44. A The answer is goal-setting; the hallmark of task-centered therapy. This is an information question.

45 . C  The social worker is required to respond to the subpoena, but is not required to release records since the subpoena was issued by the attorney and not a judge. Releasing the entire record of the therapy sessions to the attorney (answer “A”) and releasing only those records that make direct reference to the wife’s relationship with the son, not her treatment issues (answer “B”) are both not correct because the subpoena was issued by the attorney, not a judge. The social worker does not have to respond to the subpoena because the client’s records are confidential (answer “D”) is not correct because the social worker does have to respond to subpoenas but can object to releasing confidential information on the basis of privilege.

46. C The child appears to be exhibiting stranger anxiety, which typically develops at 8 months of age and appears to reflect maturation of the infant's perceptual skills such that he or she is able to distinguish between familiar individuals and strangers. In terms of test-taking strategy, given the child's age and lack of information about any other behaviors that might suggest pathology, the most parsimonious explanation for the child's behavior is stranger anxiety.

47.The cultural context presented in this item suggests that answer “c” is correct. When working with clients from an Hispanic culture, a social worker should be aware that family welfare is emphasized over individual welfare. Responding to the client’s disclosure by asking “how much money?” might convey to the client that the social worker does not approve of his sending his money to his family or that the social worker’s approval is conditional, depending on the amount of money he is planning to send.


48. C The symptom described in the question is listed in DSM as a symptom for both substance dependence and substance abuse. Specifically, the existence of physiological or psychological problems caused or exacerbated by use is characteristic of substance dependence. (A diagnosis of substance dependence also requires the presence of other symptoms, such as inability to control substance use, withdrawal symptoms, and/or marked tolerance). By contrast, social or interpersonal problems caused or exacerbated by use are characteristic of substance abuse.

49.
B
Supportive therapy (e.g., reassurance, advice-giving, mobilizing strengths and resources) is useful when objective stressors and losses dominate a client’s life. Supportive treatment may include encouraging and helping the woman to make provisions for her children (“d”); however, there is no information given in the question to suggest that this issue must be addressed first (e.g., we don’t know anything about this woman’s medical prognosis). Referral to a support group (“a”) would not be the social worker’s first intervention since this woman appears to be in crisis; it might be appropriate later on, however. Though the social worker might be able to recommend books about cancer (or more likely, books about coping with cancer), the social worker would be practicing beyond the scope of his or her license by giving the woman medical information about cancer (“c”).

50. A” - Preaffiliation (approach-avoidance), Power and Control, Intimacy, Differentiation, and Separation. The other answer options (answers “B”, “C”, and “D”) do not describe the “Boston Model” of group work.

51. C Aversion therapy (usually olfactory aversion) can be an effective form of treatment for individuals with pedophilia. In this case, however, you are treating an incestuous family system, not an individual with pedophilia. Treatments for incest and other forms of parental maltreatment reflect the view that most child abuse perpetrators are amenable to rehabilitation; their primary goal is to strengthen and maintain the family system whenever possible by providing a broad range of relevant services. Treatments for the perpetrator usually include individual therapy (i.e., for issues of low self-esteem, feelings of inadequacy or powerlessness, low tolerance for frustration or stress, poor impulse control, unmet dependency needs, and childhood history of maltreatment); marital therapy (i.e., to resolve marital conflicts, improve communication between the parents, explore dependency needs, increase the parents' ability to gratify each other's needs, help the parents recognize their respective roles in the maltreatment, and encourage greater reliance on outside support systems); and/or group therapy.

52. C When you suspect that a woman is in denial about domestic abuse (but is not in immediate danger), it is recommended that you focus initially on building a trusting relationship with her. You should allow her to move past the denial at her own pace and avoid attempting to break through it using techniques such as direct confrontation. Forcing her to acknowledge her situation may duplicate the abuse she has experienced. In addition, she may believe that her very survival depends on continuing to conceal the abuse, perhaps even from herself.

53.
C
The question says the client can afford to pay but is refusing to do so and that the social worker has tried to address the client’s reasons for nonpayment. Under these circumstances, a social worker is ethically allowed to terminate treatment if the client does not pose a danger to self or others and he has discussed with the client the potential consequences of nonpayment and termination of treatment. The social worker also should document his impressions about this situation in the client’s record.


54. B Kubler-Ross's stages, in order, are: Denial, anger, bargaining, depression, and acceptance. The statement in "b" best reflects denial, the usual initial response to learning of one's impending death. Alternative "a" is more characteristic of the second stage (anger). Responses "c" and "d" best reflect the fourth stage (depression).

55.
A
Generally, in the context of an EAP, information about an employee/client’s condition or treatment should not be disclosed without the employee/client’s consent. And confidentiality can also be breached under the same conditions that it can be breached in other contexts (e.g., danger to self, mandated reports). And, as “a” says, when the employee/client comes to therapy as the result of a referral by her supervisor, the supervisor can be given certain limited information: He or she can be told if the employee/client kept the appointment, whether the employee/client needs treatment, and whether the employee/client accepted treatment. The supervisor should not be given any other confidential information unless the social worker has permission from the employee/client.


56. D Neuroleptic (antipsychotic) drugs are used primarily to treat schizophrenia and other psychotic disorders. Although neuroleptics are sometimes used for acute aggression, they are not the treatment-of-choice for chronic forms, both because of their ineffectiveness for this problem and the potential for harmful side-effects. On the other hand, a neuroleptic is often useful during the acute phase of mania when the individual is experiencing transient psychotic symptoms. Tourette's disorder is another disorder that responds favorably to neuroleptics.

57. C  There are lots of clues for Bipolar Disorder, and it is clearly a manic episode. He is feeling great, but what about the people around him sees him differently, “irrational”.

58. D The processes of accommodation and assimilation are critical to Piaget's theory of cognitive development. These processes enable the child to progress from one stage to the next. Assimilation involves fitting a new situation or object into one's existing cognitive structure or schema, and is therefore the correct answer. Accommodation ("c") occurs when the child modifies his or her existing schema to fit a new situation or object. Equilibration ("a") refers to the establishment of a state of equilibrium through a combination of accommodation and assimilation. To Piaget, people adapt to their environment through cognitive growth, and such "adaptation" ("b") involves both assimilation and accommodation.

59. B Hiring a professional interpreter (with permission from the client) would be the most effective (and timely) way of facilitating communication with a deaf client. While the client may have a friend or a family member who can help you to communicate with her, there is no indication that the client would want a significant other to be present in her therapy sessions.

60. C Answer "c" would be the appropriate action with any client. Some gay/lesbian clients present with concerns that relate in a direct way to their sexual orientation; more often, however, gay/lesbian clients seek professional help for reasons that have nothing to do with their sexual orientation or that are related to it only indirectly. Before formulating treatment goals and a treatment plan, however, you would need to fully understand the clients' problem -- this understanding encompasses not only the way the clients define the problem as they enter treatment, but also any other issues you uncover during a comprehensive assessment that may affect the problem, but that the client are not aware of.

61. D While it is important to respect and honor other cultures' customs and values, there is a therapeutic issue that needs to be addressed in this case. Response "d" incorporates being aware of cultural differences, as well as addressing the relevant clinical issue. You are inviting the family to talk about their perceptions of time, which would encourage a dialogue, provide you with important information, and, ideally, offer a way of correcting the problem. While the awareness mentioned in "a" is an important prerequisite, answer "a" does not indicate how you would handle this situation. Although there is some truth to the statement in "b," you do not have enough information to conclude that this is true for this family; in addition, "b" does not indicate how you would address the present situation with the clients. "C" would be premature: If, after talking to the clients about this issue, you determined that their needs would be better served by a social worker from their own culture, you could make this referral at that time.

62. A The information presented in this question suggests that you need to interpret this situation within a cultural context. Only answer "a" describes what may be a typical reaction for an Asian individual. A traditional Asian individual may have difficulty discussing family matters with an "outsider" (such as a social worker), especially matters concerning his parents and other elders. In addition, a traditional Asian individual is likely to emphasize interdependence and responsibility to the family (rather than independence and autonomy) and respect for his elders (rather than egalitarian values).

63. B Clinical issues that commonly arise when working with elderly clients include finding meaning in life, developmental issues (e.g., Erikson's integrity vs. despair stage), coping with changes and loss, unfinished business (coming to terms with the past and finding meaning in the present, anticipatory loss of one's own life), and intergenerational issues (e.g., conflict within the family). In this case, the man is complaining of feeling depressed, isolated, and useless, suggesting that it would be most useful to help him develop or reestablish a sense of fulfillment or purpose in his life.

64. D Although you want a child to feel free to express herself in play therapy, it is also important to provide structure and set limits so that, for example, the child does not get hurt. Alternative "c" probably would be countertherapeutic because the goal of play therapy is to help a child express her feelings -- the unruliness may be an expression of feelings.

65. C As its name implies, strategic family therapy involves the use of "strategies." Of the responses given, alternative "c" is the best example of the techniques used by a strategic family therapist. The therapist is assigning the couple a task that will help them face their own conflicts. The other choices are not characteristic of strategic family therapy.

66. B An important issue here is whether or not the content of a dream is enough to constitute a reasonable suspicion of child abuse. You need to also consider a social worker’s ethical obligation to maintain his or her client’s confidentiality and provide effective therapy. Option “b” is the best of the four answers: The social worker needs to create a safe atmosphere where Jeremy can feel comfortable exploring the meaning of his dream. Because we have no clear indication that the social worker should suspect actual abuse at this time (see below), “a” does not address the client’s needs. Rule out “c”: The client is taking a risk and confessing a dream that is disturbing. Incestual dreams, similar to homoerotic and sadomasochistic dreams, are not uncommon, though few people are open to discussing them. Having seen Jeremy for six months, the social worker would have an idea of whether he has a propensity to be abusive; the social worker would want to be aware of the possibility that abuse is taking place, but a dream alone does not raise a reasonable suspicion of abuse. Although a client-centered approach is appropriate, as “d” suggests, it would not serve a useful purpose for the social worker to review the limits of confidentiality right now. Doing so would most likely shut the client down and hold him back from revealing any more information.

67 D  you cannot testify legally and ethically. Choice a is incorrect. Choice B is Technically correct, but not as good as choice D. Choice C is incorrect.

68. A The goal of "prescribing the symptom," a paradoxical technique, is to help family members change their dysfunctional rules. Choice "a" describes the purpose of prescribing the symptom, which involves asking the family to continue or even exaggerate what it is doing in order to undermine the family's resistance to changing that behavior. Alternative "b" is a better description of the technique known as "relabeling." Response "c" is simply another way of stating alternative "b" and, therefore, is a better description of the technique known as relabeling. Alternative "d" is the goal of Bowenian family therapy, which does not typically use prescribing the symptom as a therapeutic technique.

69. B. This is the best answer offered. Play therapy would enable the social worker to explore the problem and what might underlie it, including potential child abuse, in a relatively relaxed and non threatening setting using materials that are famililar to the the boy. A,C,D involves direct questioning or confronting the boy or his mother about sexual abuse would be less appropriate for several reasons. First, the social worker has a relatively small amount of information (only one indicator) to support a suspicion of child abuse in this case. Second, even if the social worker had good reason to suspect child abuse, the assessment approaches listed in these answers are not recommended ; e.g open ended questions tend to be more effective than close-ended questions when assessing child maltreatment problems. Third a therapist must prepare a child (e.g., develop rapport with him) before raising the topic of child abuse with him.

70 D”- Neglect. While neglect is not often thought of as abuse, neglect is the most commonly reported type of elder abuse. Verbal abuse (answer “A”), emotional abuse (answer “B”), and physical abuse (answer “C") are incorrect answers. While verbal, emotional, and physical abuse are all possible forms of elder abuse, neglect is the type of elder abuse that is most commonly reported.

 71 D” - Solution-Focused. The “miracle question” often serves as a catalyst for the client to construct a more concrete vision of a preferred future for himself. Psychoanalytic (answer “A”), Humanistic (answer “B”), and Task-Oriented (answer “C”) are all incorrect answers because the technique described in this scenario is not strongly associated with these other therapies.

72. C To answer this question, you must recognize that the client has multiple problems; therefore the first step, before recommending any form of treatment, is to partialize, or set priorities, with the client. Alternative "c" (but not "a") indicates that the social worker would encourage the client to participate in determining which problem they will deal with first.

73.
B
The Education for All Handicapped Children Act of 1975 (P.L. 94-142), which is now codified as the Individuals With Disabilities Education Act (IDEA) of 1990 (P.L. 101-474), requires that each identified child with a disability be evaluated and provided with an educational plan to meet his or her unique needs. This plan must be developed before the child is placed in any educational program. The overriding purpose of the law was to guarantee that all school children with special needs would receive an appropriate free public education, and a key feature of the Individualized Educational Plans (IEPs) required by this law is that a student’s plan must provide him or her with the “least restrictive environment” at school. Parents, teachers, and school counselors are members of a child’s Individualized Educational Plan (IEP) team, and parents and teachers can appeal decisions regarding the best classroom placement for the child; the final decision on this matter, however, is made by the local school committee.



 74.
A
It sounds as though this client might be having panic attacks. Before concluding this, however, it is necessary to rule out a medical illness underlying the symptoms. Symptoms of hyperthyroidism include excitability, nervousness, confusion, higher metabolism, body temperature, and heart rate, increased appetite with weight loss, fatigue, and insomnia. Untreated diabetes mellitus (“c”) is associated with apathy, confusion, and mental dullness, increased appetite with weight loss, polyuria, polydipsia, and increased vulnerability to infection.


75. D This question is about narrative therapy, a post-modern approach in which the therapist collaborates with clients to construct a new "story" about their experience; the new story is presumed to offer greater opportunities for growth and solutions than the versions the clients are currently locked into. Narrative therapists believe that objective reality cannot be known (i.e., all "knowing" requires interpretation) and that the meaning people attribute to events in their lives determines their behavior. Narrative therapists help clients to identify "unstoried" parts of their experience ("unique outcomes") and then to develop new meanings around these outcomes. The initial step is to separate clients from the problem: This entails "externalizing" (objectifying) the problem and then "mapping" its influence in the clients' lives and relationships. Each client's description of the problem and its effects is considered "privileged"; i.e., while family members may have common definitions of problems, each client is presumed to experience a problem and its effects in a unique way. Answers "a," "b," and "c" are more straightforward, information-gathering questions, which might be asked in any standard assessment situation. Answer "d," on the other hand, implies that each family member's unique experience of the problem is important.

76. C Option “c” is the best answer to this question: American Indians tend to be more comfortable with an indirect gaze both when listening and when speaking to other people, including their therapists. Option “d” is the second best answer: Asian individuals do tend to avoid direct eye contact when listening and speaking to certain individuals, including high-status people or those in positions of authority; this preference may carry over into the treatment setting. Among Caucasians (“a”), it is considered respectful to use direct eye contact when both listening and speaking. And, although African-American individuals (“b”) tend to avoid direct eye contact when listening, they usually prefer to make eye contact when speaking (D. Sue et al., “Counseling the Culturally Different,” John Wiley & Sons, New York, p. 79, 1999).

77. A Ellis's RET assumes that irrational ways of thinking underlie dysfunctional behaviors. Alternative "b" (dysfunctional behaviors are the result of incongruence between self and experience) is the underlying premise of Rogers's client-centered therapy. Alternative "c" is not the best response given because, although RET views irrational thoughts as determiners of dysfunctional behaviors, the notion of "automatic thoughts" is more characteristic of Beck's theory of and treatment for depression. Response "d" (dysfunctional behaviors are the result of a lack of "awareness") is the underlying premise of Perls's gestalt therapy.

78. A Minuchin's structural approach to family therapy regards dysfunctional families as being either "enmeshed" or "disengaged." In an enmeshed family, boundaries between family members and subsystems are too diffuse so that family members are overly dependent and close; in a disengaged family, boundaries are overly rigid so that family members are too isolated from one another. Minuchin's structural family therapy can be characterized as "manipulative, unyielding, and crisis provoking" (Goldenberg & Goldenberg, 1985). It uses a number of techniques to restructure a family's transactional patterns including marking boundaries, escalating stress, utilizing symptoms, and manipulating mood. When using the latter technique, the therapist might imitate, in an exaggerated way, the family's style in order to point out their dysfunctional patterns. One criticism of Minuchin's approach is that it does not emphasize fostering insight and understanding; therefore, alternative "b" is incorrect. The genogram ("c") was developed by Bowen and is used primarily by family systems (Bowenian) therapists in order to clarify a family's multigenerational relationship systems. The therapeutic double-bind ("d") was an outgrowth of research on the pathological double-bind conducted by Don Jackson and his colleagues, and is used primarily by communications family therapists.

79. D Yalom views conflict in group therapy as a natural phenomenon and considers it to be a useful therapeutic tool if handled correctly. According to Yalom, the therapeutic use of conflict in group therapy involves two stages: Experience (affective expression) and understanding of that experience.

80. B Choice "b" is the best answer by process of elimination. With any client who has been raped, you should emphasize listening and providing emotional support, but this intervention is not listed among the alternative answers. Certain medical and legal procedures also should be carried out after a rape (e.g., medical care, gathering of evidence for possible prosecution). While these procedures should not be emphasized at the expense of providing the client with emotional support, it is appropriate to help this woman explore her resources, including the importance of seeking medical attention (e.g., she may have sustained injuries or been exposed to a sexually-transmitted disease). Rape is not a mandated report, unless the victim is a minor. Therefore, you could not call the police ("a") without permission from your client. Moreover, she has said that, at this point, she does not want to press charges. "C" is not appropriate, since you should not probe for details about the rape; you should allow the client to reveal these at her own pace. Last, even if the woman wanted you to speak with her husband, contacting him ("d") would not be your first intervention. Moreover, it is not your role to remind clients or their family members about their "community responsibility."

81.
C
The only appropriate answer is “c” since the social worker no longer owns this apartment. Answers “a” and “b” reflect dual relationships that might end up harming or exploiting the client, and “d” suggests that the social worker would recommend a living situation that does not meet a basic need this client might have — he is described as “low-income,” suggesting that it would be important for him to have access to public transportation.


82. B The answer is having the parents work through the disagreement in an appropriate manner. What created the anger is not nearly as important as learning to resolve conflicts in appropriate ways. The tendency of these parents to vent their anger has gotten them into trouble in the first place. Insisting that the parent stop arguing does not help him or her to learn appropriate ways of resolving differences.

83. B Process Recording is a system for recording social worker-client interactions over the course of treatment. Contrary to “a,” it does not require verbatim documentation of worker-client interactions. Instead, the case record begins with “face-sheet” factual information about the client and relevant social, physical, and economic data. A description of the presenting problem follows, including all data the worker has collected about the client’s problem. The record then contains the worker’s description of therapy objectives, obstacles to achieving the objectives, means to reaching them, and, when relevant, a contract signed by the worker and client. Next, the record contains entries for each contact the client makes with the worker and agency, including not only face-to-face contacts, but also telephone contacts and contacts with family members. These entries include the date and time of the contact and a summary of the information obtained, including any subjective impressions derived by the social worker.

84. C In Transactional Analysis (TA), "scripts" are the life plans that people live by; they result primarily from early messages and injunctions the child gets from his or her parents. "Games" are recurring transaction
83. B Process recordin s between people that have a concealed motive. An initial process in TA is the identification of a client's life scripts and the "games he or she plays" that maintain those scripts.

85. B According to Murray Bowen, a high degree of fusion (emotional "stuck togetherness") is the source of family dysfunction. The goal of Bowen's form of family therapy, therefore, is to help family members become less fused, or more differentiated (individuated). In contrast to many other family therapists, Bowen's form of therapy typically does not involve seeing all of the family members. Instead, Bowen often worked with the most differentiated family member on the assumption that, when that member changed in a positive way, his or her change would motivate other family members to move toward greater differentiation. Alternative "a" is the opposite of the correct answer; as noted, followers of Bowen would likely work first with the most differentiated family member. Alternative "c" is incorrect because, in contrast to many other family therapists, Bowen typically worked with the two most significant adult family members, usually the parents, or the parent who was the most differentiated, even when the "identified patient" was a child. Alternative "d" is not a technique used by Bowen.

86. A In order to make a recommendation based on current and complete information, you should have the child retested to see if special education is appropriate. If you determine that it is, additional follow-up actions might be required given the nature of the situation. Therefore, alternative "a" is the best response of those given. Alternative "b" is incorrect because the student's welfare is of greatest importance in this situation, and referring the teacher for therapy would not provide a rapid solution to the problem. Response "c" is incorrect because previous test results may not be an accurate indication of the child's current needs. Alternative "d" assumes that the principal and teacher are acting inappropriately; however it is possible that their objections are well-founded (i.e., the student may not need to be placed in a special education class).

87. B This question should have been fairly simple. The names of the defense mechanisms act as a clue to their manifestations. Choice "b" is an example of projection, not displacement. Displacement involves shifting an undesirable impulse to a more acceptable and less threatening object. Reaction formation involves warding off an undesirable impulse by overemphasizing its opposite. Sublimation involves altering unacceptable impulses by channeling them into acceptable, even admirable, behaviors. Regression is what it sounds like -- a return to an earlier stage of development.

88. D The various cognitive therapies emphasize the importance of cognitive processes as determinants of behavior. They assume that behavior and emotions result largely from one's appraisal of a situation, and that, because one's appraisal is influenced by beliefs, assumptions, images, and self-talk, these cognitions should be the targets of therapy. Compared to other forms of therapy, cognitive therapy more directly challenges the client's faulty assumptions, beliefs, and thoughts. Delusions are false beliefs that cannot be corrected by reasoning and that are not consistent with the client's intelligence or cultural background. Cognitive-restructuring techniques can sometimes be effective for delusions; cognitive-retraining techniques, however, should be used only after the therapeutic alliance is strong enough for you to question and challenge the client's faulty beliefs.


89 “C” - Alan likely has Encopresis. Encopresis is a pattern of expelling feces into places deemed inappropriate and would therefore not apply in this situation. To make a diagnosis of Enuresis, which is a specific Elimination Disorder, a child must have reached the chronological or developmental age of five. Inappropriate urination must occur at least twice a week for three months or the frequency of inappropriate urination must cause significant distress and interfere with the child’s school or social life. Finally, the behavior cannot be caused exclusively by a medical condition or as a side effect of medication. The other three answer options (answers “A,” “B,” and “D”) are all true statements regarding Alan’s situation, and the question asks which answer option is NOT a true statement. Because stressful life factors could also affect bed wetting, the social worker should continue to assess the situation in considering other factors.


90. C Research suggests that only one in ten cases of spouse abuse is reported and that, in therapy, few couples "present themselves to the therapist with abuse as the presenting problem" (Mack, 1989). Because directly confronting couples about abuse may elicit denial, a more indirect approach is preferred. As noted by Mack, this approach "is not accusatory and does not immediately focus on abuse, but provides an opening for the therapist should the data gathered point in the direction of abuse" (p. 199). Alternative "d" would be the next step after abuse has been revealed. The first goal in treatment in cases of spouse abuse is to the stop the abuse, and a "no-violence contract" has been found helpful for achieving this goal. Alternative "a" is incorrect because the issue of abuse must be identified and dealt with early in treatment to ensure that it is stopped as soon as possible. Thus, waiting for the husband or wife to bring up the issue of abuse would not be considered appropriate in conjoint therapy or any other type of therapy. Alternative "b" is incorrect because, although some conjoint therapists do see husbands and wives individually, especially during the initial sessions of therapy, conjoint therapy usually involves seeing both parties (not just the wife). In addition, while most experts do recommend individual, rather than conjoint, therapy in cases of spouse abuse, so far in this situation, you have yet to confirm the existence of spouse abuse.

91. B In this situation, the child's behavior is increasing following the parents' intervention; in other words, the child is being reinforced. Reinforcement, by definition, increases a behavior. This is positive reinforcement, not (as in choice "c") negative reinforcement. Negative reinforcement involves the removal of a stimulus; here, a stimulus is being applied.

92. D Reinforcement involves applying or removing a stimulus in order to increase a behavior, while punishment involves applying or removing a stimulus in order to decrease a behavior. Alternative "a" is incorrect because both reinforcement and punishment involve stimuli and responses. Although reinforcement and punishment are often associated with, respectively, pleasant and unpleasant stimuli, from a strict behavioral point of view, the distinction in alternative "b" is incorrect. As noted above, reinforcement and punishment can both involve either the application or removal of a stimulus, so the distinction in response "c" is also incorrect.

93.  C  This child had problems with attachment related to her home situation (neglect of all sorts), and it is clear from her behavior that she is disinhibited (the excessive familiarity with strangers).


94. C During the termination phase of therapy some clients (even those for whom treatment has been successful) experience the loss of support from the therapeutic relationship as uncomfortable. This may produce a regression or a reappearance of the presenting symptoms ("flight into illness"). In psychodynamic terms, this would be viewed as a defense mechanism. Freud believed that anxiety serves to alert the ego to danger, including danger arising from a conflict between the id's impulses and the demands of reality or the superego and danger arising from an actual threat in the environment. When the ego cannot ward off danger using rational, realistic means, it may use one its defense mechanisms. The defense mechanisms function to defend against unacceptable unconscious impulses or drives (i.e., to keep them from reaching awareness).

95.
B
Partners of rape victims often have conflicted feelings. Being in a group with others facing a similar situation would help this client feel normal and reduce his sense of isolation. The wife may have too many individual issues to be available for couples therapy, so “d” is a poorer choice. Answer “a” is appropriate, but “b” responds more directly to the client’s expressed needs. In the context of this question, which asks for the best approach to treatment, “c” is a poor choice because it doesn’t describe what kind of treatment you would offer this man.
.

96. B The client has lost functioning in his hand and the physician has been unable to find a biological cause. Conversion disorder involves a symptom or deficit that affects voluntary motor or sensory function, cannot be fully explained by a general medical condition, the effects of a substance, or another mental disorder, and is not produced voluntarily or controlled by the patient. Instead, the symptom or deficit is presumed to be related to psychological factors – in this case, probably the fact that the man killed people when he fired a gun with this hand. Rule out “a” and “d” because we have no evidence that the client is intentionally producing or faking his symptom: In factitious disorder, the patient intentionally produces or simulates physical or psychological symptoms out of an intrapsychic need to adopt the “sick” (patient) role; and in malingering, the patient deliberately fakes or exaggerates a symptom in order to avoid a responsibility or obtain an external reward. And although PTSD (“c”) is often associated with having been in combat, this client has no signs of PTSD (e.g., no anxiety, no re-experiencing of events from the war).

97. D Rogerian therapists tend to de-emphasize both diagnosis and assessment. They usually reject diagnosis on the ground that it places the therapist in a superior, authoritarian role that can impede the development of autonomy and self-actualization in the client.

98.
B
The parents want help placing their son in a psychiatric hospital, apparently because he is gay. Option “b” is the best answer because it directly addresses the parents’ presenting concern. The social worker would want to educate them about the purposes and appropriate uses of psychiatric hospitalization, discuss homosexuality with them, and explore and allow them to vent their feelings. Option “a” omits any mention of the parents’ feelings, which seem to need the social worker’s attention. The referral in option “d” is premature, given the parents’ current strong feelings; in addition, “d” fails to address their desire to hospitalize the boy. Option “c” is clearly incorrect, because there is no good reason to believe that this boy is simply passing through an “experimental phase.”

99.
A
This question required clinical reasoning. The client (who is depressed but not a danger to himself) has been seeing the social worker for five months without progress and has refused repeatedly to see a psychiatrist for a medication evaluation, even though doing so might help him. It’s possible that he is not ready to change or that this social worker is not the right therapist for him. Therefore, discussing termination and referral is consistent with a social worker’s ethical obligation to discontinue treatment when it’s clear that a client is not benefitting from his or her services. Because the social worker also has an obligation to respect a client’s right to and need for self-determination, the social worker cannot force this man to see a psychiatrist; however, this does not mean that the social worker should simply continue to treat the client as before (“d”), since he doesn’t appear to be benefitting from the social worker’s services. And, we can rule out inpatient treatment (“b”) because the question says that the man does not pose any danger to himself at this time.

100. C Knowing that cognitive therapists emphasize the identification and replacement of dysfunctional cognitions would have helped you select the correct answer to this question. Alternatives "a" and "b" are more characteristic of Bandura's approach, which incorporates cognitive techniques but is not a typical example of cognitive therapy. Bandura's treatment goal is to increase a client's self-efficacy beliefs, which is not a technique characteristic of cognitive therapy. Response "d" is also not characteristic of the cognitive approach.

101. B Minuchin's structural family therapy is based on systems theory and views a family member's symptom as a factor that helps maintain the family's homeostasis. A primary goal of therapy is to disrupt the family's homeostasis in order to help the family change their dysfunctional patterns. Alternative "a" is incorrect because Minuchin and his followers typically join with the family by imitating its manner, affective style, and communication patterns. Minuchin and his followers do just the opposite of alternative "c"; i.e., they encourage confrontation in order to disrupt the family's current homeostasis. Response "d" is a characteristic of Murray Bowen's approach to family therapy, not Minuchin's.


102. B A social worker must consider relevant cultural/religious differences when evaluating whether her clients’ behavior constitutes child abuse or neglect. Due to the severity of this boy’s medical condition, however, his parents’ unwillingness to follow the doctor’s urgent treatment recommendation would be considered a form child endangerment, or neglect of his medical needs.

103. B In gestalt therapy, the primary goals are increased awareness and integration of all aspects of the personality. That is, a primary goal of gestalt therapy is to integrate all aspects of the self; i.e., all aspects of the mind and body. Alternative "a" is incorrect because, in gestalt therapy, the focus is on the present. In fact, Perls argued that nothing exists but the "now." Response "c" is the goal of Reality Therapy. Alternative "d" is not a goal of gestalt therapy. In fact, internalization of the external is viewed as a potential source of pathology by gestalt therapists.

104. D Minuchin distinguishes between two types of disturbed boundaries: Enmeshed and disengaged. In disengaged families, there is a lack of emotional involvement. Family members not only avoid expressions of love and concern toward one another but also avoid conflicts and the expression of anger. When family members are enmeshed, they are overly involved with one another, so alternative "a" would be characteristic of an enmeshed family. Disengagement entails a lack of involvement, so response "b" would be characteristic of a disengaged family. Alternative "c" is incorrect because one benefit of enmeshment is that family members may receive a great deal of attention and concern.

105. C In his book, "The Theory and Practice of Group Therapy," Yalom (1985) identifies the types of people who are and are not likely to benefit from group therapy. He argues that heterogeneous outpatient groups are inappropriate for sociopaths (although they may benefit from homogeneous groups in inpatient and other controlled settings). According to Yalom, a person suffering from anhedonia or a workaholic is a good candidate for group therapy. He also believes that disorders related to social interactions are amenable to group therapy.

106. D Both legally and ethically, you must take reasonable steps to prevent a threatened suicide by a client. To determine what these "reasonable steps" should be, you should evaluate the degree of risk posed by the client and determine the client's ability and willingness to comply with your recommendations. Mobilizing the support system is a reasonable step with a client expressing suicidal ideation. For example, you might suggest that the client temporarily stay with friends or relatives, arrange to see the client more often, and have the client touch base with you over the phone at regular intervals. In addition to providing support, family members or friends can also be a source of logistical help, by keeping an eye on the client and/or removing all weapons from the client's home. (Though not mentioned in the answers, this intervention should also entail developing a no-suicide contract in which the client promises not to kill himself or herself for a specified period of time and promises to contact you or a suicide hotline whenever he or she feels an impulse to commit suicide.) Calling the parents ("c") is just one way of mobilizing a client's support system, so "d" is a better answer. If this client has unresolved anger, "b" might be an appropriate long-term treatment goal, but it is not a first intervention with a suicidal client.

107. C Negative reinforcement occurs when the removal of a stimulus following a response increases the occurrence of that response. In the situation in choice "c," restrictions are removed following a behavior in order to increase the behavior. Alternatives "a" and "d" are examples of negative punishment; i.e., something is being taken away following a behavior apparently in order to decrease the behavior. Response "b" is an example of positive reinforcement

108.
B
The first thing to do is assess why the boy is interested in this information. Answers “a” and “d” would violate the boy’s confidentiality, unless he had given the social worker permission to discuss this matter with his biological family or foster parents. It might be perfectly appropriate for the social worker to help the boy attain the information he wants (“c”), but first the social worker should discuss the boy’s interest with him as a means of increasing the boy’s self-awareness and the social worker’s understanding of him and his needs.

109.
B
The focus of the court’s inquiry in this case is on the assessment of problematic alcohol use (not antisocial conduct, which is mentioned in “c”); also, individuals are at higher risk for alcohol abuse/dependence when they have a family history of this problem. A genogram can be used to identify behavioral patterns in a client’s immediate family, including whether or not other family members have or have had substance use disorders. Although constructing the genogram would require the client to remember aspects of his childhood, assessing long-term memory (“a”) is not a purpose of this assessment tool — e.g., a client’s “inability” to remember information about his family could be due to resistance, rather than memory deficits. Similarly, while the social worker is likely to assess the effects of alcohol use on this client’s interpersonal functioning (“d”), a genogram is not used for this purpose.


110. A According to Freud, the defense mechanism "projection" involves attributing one's own unacceptable impulses to another person. Answer "b" is generally correct, but it is a layperson's description of what is happening. Rule out "c" because the man's desire is conscious (expressed), rather than unconscious. According to Freud, Oedipal feelings ("d") refer to a young boy's erotic interest in his mother and resulting feelings of rivalry toward his father.


111. A In group therapy, resistances can take several forms and occur most often in the early-middle stage of therapy when members try to establish their preferred position in the group. By definition, resistances are covert efforts to undermine change; thus, they are best handled by being brought out into the open. Discussions should focus on the here-and-now and deal with what the resistances imply. Alternative "b" is wrong because, while the developmental process of the group might be of interest to some members, this approach would do little, if anything, to deal with resistances. The strategy in alternative "c" would probably engender resistances in greater number and intensity. Response "d" is incorrect because capitulation to resistances amounts to reinforcement of them and is likely to increase their occurrence.

112. A A social worker is ethically required to take reasonable steps to prevent the suicide of a client, and only answer "a" describes an appropriate way of doing so in this case. These "reasonable steps" are determined by the level of risk posed by the client. In this case, the client is depressed and has a detailed suicide plan, suggesting a high level of risk. If you believe that a client is a high risk to himself and he refuses less aggressive means of protective intervention (e.g., voluntary hospitalization), you need to take more aggressive protective steps, such as initiating involuntary hospitalization. Answers "b" and "c" might be appropriate interventions for a client who is at less risk for harming himself.

113. B  is the best answer, hopefully, you have already discussed options and your response with Martha should you be in a similar situation. Choice A is incorrect. Choice C is partially correct. You cannot legally even acknowledge that you know Martha to Mrs. Jones, much less give her any information. Choice D is incorrect .

114.
B
The social worker’s “next” step is to complete the biopsychosocial assessment of this client. The question describes two components of a comprehensive biopsychosocial assessment – “psycho” and “social.” Answer “b” describes the third component – “bio.”


115. C. You have to know your medications, Depakote is one of the major ones.


116.
B
In general, treatments that target the individual (e.g., pharmacotherapy, individual therapy) are insufficient in treating conduct disordered children, and most approaches emphasize parent training and other techniques that target the family, the school, and the community. Parent training is often combined with operant techniques at home and at school.


117. B In this case, the social worker’s client is the child, and the purpose of the home visit was likely to collect information to facilitate his work with the child. The social worker has apparently discovered some disturbing information that probably affects the child, as well. However, given the mother’s obvious reluctance to talk about the problem, the best action would be to continue helping her child. Doing so would allow the social worker to build a relationship with the mother and, after a while, she might come to trust the social worker. Although the action in “a” might protect the mother and child, it is not appropriate because it states that the social worker would call the shelter, and would do so despite the mother’s reluctance to bring the abuse out into the open at this time. (A good answer would have been that the social worker should give the mother information about shelters, but we’re not offered that choice.) Option “d” is not best choice because the woman may feel threatened or alienated if the social worker confronts her. If the social believed that there was imminent danger, then he could be more active in encouraging the woman to acknowledge the abuse, but otherwise, a therapist should usually avoid confronting a battered woman with the “reality” of her situation because she may feel that this replicates the abuse by her husband. And, because the question clearly indicates that the mother is reluctant to admit she is being abused at this time, “c” could also alienate her, so that the social worker might never be able to help the family.

118. B Option “b” is “most” likely to be true. First, as “b” says, these clients may not view the man’s alcohol use as a problem, even if the social worker does. Substance abuse is one of the most significant problems faced by American Indians, and experts (e.g., Sue & Sue, 1999; Manson, Tatum & Dinges, 1982) have attempted to explain this in a number of different ways. Many of these explanations reflect a view of alcohol use that probably differs from the one this social worker has – e.g., among American Indians, refusing a drink offered by someone else is viewed as an act of personal autonomy that disrupts group harmony; drinking alcohol is a social event; and drinking alcohol is an accepted practice and is encouraged among family members. In addition, alcohol use may allow American Indian individuals to release feelings that are otherwise kept under control (e.g., frustration, boredom). Second, Sue and Sue (1999) say that when working with American Indian clients, therapists often need to address the clients’ basic needs first, such as problems stemming from poverty (e.g., inadequate food, shelter, child care, or employment). This is recommended, in part, because many traditional American Indians are more oriented to the present than the future. Clients who are oriented to the present tend to prefer interventions that work in the here and now.

119. D This is the best answer offered. There is no information given about where the father is living and the question does not indicate that you have had an opportunity to observe or evaluate him yourself. Before advising the children about whether or not their father needs to be placed in an independent living facility or recommending any other services, you would need more information about his level of daily functioning. Because the children are your clients, not the father, you do not have permission to do an in-home assessment at this time; you could, however, suggest to the children that this would be the best way for you to gain an understanding of their situation and determine the best way to help them. You might then be able to attain permission to evaluate the father. Giving the children literature on independent living facilities ("a") suggests that you agree with their decision and, given what we know about the father, such a decision would seem to be premature. There is no information given that would lead you to conclude that the father needs a guardian ("b").

120.C Therapy groups may be either “closed” or “open.” Closed groups begin and end with the same members and usually have a preset termination date. Open groups permit members to join and leave the group at different times. Some authors also identify open groups as those without a set number of sessions or a preset termination date.

121. C This is pretty straightforward. The boy's welfare is your first concern, and you'd want to make sure that he is not at risk for suicide. Suicide threats and ideation should always be taken seriously. Before notifying his parents, however, you'd want to determine how serious his thoughts about suicide are.

122. C This is essentially a "common sense" question. Obviously you can't force a client to continue seeing you, but you should discuss your reservations about his termination. If you believe it would not be good for the client to quit at this time, you should discuss this with him. If he still wants to leave, then you'd want to assure him that he can come back and/or give him appropriate referrals.

123. A Respite services would provide temporary care for the children, thereby allowing the woman to pursue other activities to improve her situation (including, for example, vocational training or a support group). Note that there is no information given to suggest that the woman needs to attend a parenting class.

124. D The incorrect answers are not bad interventions, but the social worker’s priority is to prevent the girl from having any more contact with this man, and “d” would be the most direct and effective way of attempting to do this. When a social worker believes that a child is being persistently pursued by an adult over the Internet, he should encourage the child and her parents to report the situation to the police or the FBI (the FBI has a task force that investigates cases of child abuse over the Internet). If this girl said that she didn’t want the social worker to tell her parents about her “friendship” with this man, the social worker would be justified in breaking her confidentiality because doing so appears necessary to protect her. The social worker should tell the girl that he needs to discuss this situation with her parents; and with the parents he should discuss the option of notifying the police. He should also talk with the parents about how to monitor the girl’s use of the Internet more closely and how to better control the content she can access when she’s online..

125. A The problem-solving model emphasizes deficiencies in the client's problem-solving skills; specifically, a lack of motivation, capacity, and/or opportunity to solve a problem. Alternative "b" is incorrect because any social worker might modify a client's environment; this goal it is not specifically associated with the problem-solving model. Alternative "c," an emphasis on modifying the person-in-situation system, is associated primarily with the psychosocial model. Alternative "d" sounds more like the functional model, which emphasizes the individual's potential for growth.

126. A Of the available responses, only alternative "a" suggests a way of providing the client with an adjunct to psychotherapy that might improve her depression (presumably the psychiatrist would conduct a medication evaluation). In other words, the client may progress if the social worker modifies the treatment plan. Nevertheless, this question is difficult because, probably, the social worker should have recognized before now that the client might profit from taking medication. However, the other two likely alternatives ("c" and "d") are even more clearly components of the initial phase of helping with a depressed client (i.e., they would have enabled the social worker to determine a diagnosis). Answer "b" is incorrect because it suggests that the lack of progress is just the client's fault. In this context, it is better for the social worker to attempt to provide the client with additional and possibly more effective modes of treatment.

127. D The process illustrated in the question is homeostasis, which describes the tendency for a system to react in a way that will re-establish the "status quo" in the event of change. Family therapists predict that families will make systematic efforts to restore equilibrium when their balance is disturbed or threatened. Therefore, when a "sick" family member gets better, another family member may develop symptoms to restore balance in the system. Complementarity ("a") is said to exist when the tendency of one person in a relationship enhances the tendency of the other person; e.g., when a wife's dominance increases her husband's submissiveness. Wholeness ("b") is also a property of systems. It states that every part of a system is so related to the other parts that change in one part will automatically cause change in the other parts and in the whole system. In this situation, change in one part of the system (the alcoholic) has led to change in one family member. not the entire system. Equifinality ("c"), which is also a general systems concept, predicts that dissimilar causes can produce the same results.

128. C With few variations, crisis intervention is a well-defined form of treatment in which a therapist applies the same principles, regardless of his or her theoretical orientation. The therapist is directive and does not allow the client to select from a variety of treatment alternatives.

129. B The most pressing issue described in this question is the women's recently emerging difficulties with performing activities of daily living. Homemaker services are designed to help individuals remain in their own homes; they assist with daily living chores (preparing meals, cleaning, etc.) and provide transportation and some nursing services. The referrals in "a" and "d" may be given, as well, to broaden the women's support network but, from a "safety" standpoint, the resource "b" is more important.

130. D Only the intervention in "d" would enable the social worker to evaluate and begin to address the man's immediate needs and emotional state in a timely manner.

131. A The stem does not specify the nature of the client's question. In general, however, the best approach would be to have the client bring the issue up in front of the group. In the first stage of group therapy, the members tend to look to the group leader to answer their questions and to interact with one another in only limited ways. Encouraging the member to bring his question up during a group meeting would facilitate the development of group cohesion and functional behavioral norms.

132. D Unlike classical (i.e., Freudian) psychoanalysis, in which past experience is emphasized, ego psychology focuses on current problems. The other available responses are characteristics of classical psychoanalysis. Specifically: 1. Freud emphasized transference in therapy, while the ego analysts relied less on transference and more on offering the client opportunities for reparenting and building more adaptive defenses. 2. Freudian personality theory (which is the basis for classical psychoanalysis) focuses on the role of the id (sexual impulses) in the formation of personality, while ego psychology focuses on the impact of the ego. 3. While Freud conceptualized maladaptive behavior as due to intrapsychic forces, the ego analysts considered that maladaptive behavior results when the ego loses its autonomy from the id or from reality. When this occurs, behavior is no longer under the individual's conscious control.

133.A  Children generally begin to understand the concept of death between the ages of 7 and 9 years, and the concrete operational stage (Piaget) occurs between the ages of 7 and 12 years. The preoperational stage (“d”) occurs between 2 and 7 years; Freud’s phallic stage (“c”) occurs from 3 to 6 years and the genital stage (“b”) begins after puberty.

134. D In most situations, a therapist will want to get an informed consent before providing professional services. The need for an informed consent is not waived when a treatment is court-ordered (e.g., when it is a condition of probation or parole). Alternative "a" is incorrect because, in some emergency situations, it is not feasible or necessary to get an informed consent (e.g., when a client is having a psychotic episode and must be hospitalized immediately for his or her own protection). Alternative "b" is incorrect because competence to understand information about a procedure and its possible consequences is a condition for a truly informed consent. When an individual is not competent, his or her assent (rather than consent) should be obtained. Response "c" is incorrect because a client can, of course, waive his or her right to informed consent (but should be warned of the consequences of doing so).

135. A The incorrect answers are too limited. According to Barker (“The Social Work Dictionary,” 5th Ed., 2003), permanency planning is, “... a systematic effort to provide long-term continuity in dependent children’s care as an alternative to temporary foster care placements.” Efforts associated with permanency planning include facilitating adoptions (“d”), establishing clear guidelines for remaining in foster care, and helping biological families become capable of meeting their children’s needs (“c”). Workers attempt to prevent out-of-home placement whenever possible and to restore families if a child has been removed from his or her parents’ home. They also apply guidelines designed to improve out-of-home placements (“b”), such as placing a child in the least restrictive, most family-like, setting available and as close to his or her parents’ home as possible. The placement setting should also be consistent with the child’s best interests and should meet any special needs the child has..

136. B The best answer offered is "b." A social worker should consider cultural/religious differences when assessing whether or not a parent's behavior constitutes child maltreatment. Due to the severity of the boy's medical condition, however, his parents' unwillingness to follow the doctor's urgent recommendation would be considered a form child endangerment or neglect of his medical needs. Answers "a" and "c" might be good choices if this were not a medical emergency.


137. B A social worker is legally and ethically required to make a report of known or suspected child abuse to the appropriate authorities. The action in alternative "b" is consistent with state laws and the provisions of the Ethics Code. Choice "a" is incorrect because a social worker must make a report even if the abuse occurred in the past and it seems unlikely that it will occur again. This makes sense, since the victim of the abuse is likely to need of physical and/or psychological help. Alternatives "c" and "d" are incorrect because reporting the incident to the child's parents would be an unauthorized breach of confidentiality and not in accordance with state laws.

138. D When answering questions related to confidentiality or privilege, the best answer is usually the most conservative answer. A social worker's first response to a subpoena duces tecum should be to seek legal advice to determine what his or her responsibilities are. Usually, the most appropriate action will be to assert the privilege not to release information about a client and then to release the information only when the client has authorized such release or when the court has ordered the release.

139. C The Ethics Code requires social workers to cooperate with other professionals but also mandates that they provide others with confidential information only when the client's consent has been obtained. Before releasing confidential information, the social worker should obtain a signed consent from the girl's parents. Alternatives "a" and "b" are incorrect because providing information to the counselor without the parents' consent would be a violation of confidentiality. Although releasing only relevant information is a good and ethical practice, no information should be given without first obtaining the appropriate consent. Alternative "d" is incorrect because, although a social worker has an obligation to ensure that information is not misinterpreted by others, he also should cooperate with other professionals by supplying them with needed information after an appropriate consent is obtained.

140. C Social workers must recognize the boundaries of their competence in order to make sure their clients receive the best possible treatments. The action in alternative "c" is most consistent with the requirements of the Ethics Code. Response "a" is incorrect because it would not be in the best interests of the client to use an alternative treatment simply because Karen is not experienced in providing a more appropriate treatment. Choice "b" is incorrect because merely reading the literature on the treatment would not give Karen sufficient expertise to provide the client with that treatment. Alternative "d" is incorrect because, if a social worker knows that another treatment would be most effective and the treatment is outside the scope of her competence, an appropriate referral would be the best course of action.

141. B Although the specific details of child abuse reporting laws vary from state to state, the situation in answer "b" would always be reportable because one of the parties is under the age of 14 and the other is not: Sexual activity with a male or female under the age of 14, whether or not the activity is consensual, is considered a form of child abuse. Non-coercive sexual activity involving two minors under the age of 14 (answer "a"), on the other hand, is not reportable in most states.

142. C This is fairly simple. Since this is an ethics question, you can eliminate responses "a" and "b" because they deal more with a clinical issue. Alternative "c" is a better answer than response "d" because it more clearly takes the welfare of the client into consideration. It is also the more conservative answer, and a good strategy for ethics questions is to pick the more conservative course of action. Alternative "d" comes too close to being client abandonment.

143.  A  Answers “b” and “c” are possible, but “a” is correct because it is consistent with Piaget’s theory of cognitive development, which is alluded to in the question. A preoperational child’s thought is limited by centration, which is the tendency to focus on just one aspect of a situation or object. Therefore, this child may understand that his parents are gone, but miss the fact that they will be back in a few days.

144. C Of the choices given, "c" is the best answer. The incorrect answers would reveal the nature of your professional relationship with this person and, thereby, violate her confidentiality.

145. D From an ethical standpoint, the correct answer is "d." Social workers are encouraged to provide appropriate professional services in public emergencies to the greatest extent possible and to place service to others above their own self-interests. In addition, the NASW Code of Ethics states, "social workers should not engage in solicitation of testimonial endorsements from current clients or from other people who, because of their particular circumstances, are vulnerable to undue influence."

146. A This item goes to the worker/client relationship. We never consider a client or a relationship hopeless (goes against Social Work values), and we refer when it is in the client's best interest, not for worker comfort.

147.C Hospice care is for people at the end stage of terminal illness, when curative treatments are no longer available and death is near. It provides psychological, social, and medical care, and emphasizes the quality, rather than quantity, of life. Patients are provided with holistic treatments for pain control, including physical, psychological, and spiritual pain. Option “d” is incorrect because people accepted into hospice care must have a family member or friend who is willing to be designated as a primary caregiver. Hospice care emphasizes the involvement of loved ones in the care of the patient, and supports caregivers by providing respite and convalescent care. Answer “a” describes home health aide services. Home health aides are health care workers who provide homemaker services and personal care to people who are disabled or recovering after discharge from a health care facility.

148.
C
We don’t seem to have enough information to choose an answer: For example, does “Jack” live at the juvenile detention facility? Does the client still live at the facility? What does the client mean by the words “get him.” The answers to these questions, if we had them, would help us to determine whether “Jack” is in imminent danger of being physically harmed by the client. Whether this is a “Tarasoff” situation or not, however, it would be appropriate, as well as important, for the social worker to inform the client about the limits to confidentiality and a social worker’s related legal obligations
.

149. B  All four answers are side-effects of the antipsychotic drug Prolixin (Fluphenazine). Seizures, however, though they are rare, are the most dangerous side-effect associated with this medication. Physicians advise patients taking Prolixin to stop taking the drug and seek medical attention immediately if they experience a seizure. Other rare side effects of Prolixin include low white blood cell count, tardive dyskinesia, racing heartbeat/palpitations, tremors, dizziness, weakness, sexual problems, restlessness, skin rash, and reduced urinary output. More common side effects of Prolixin include lethargy/sleepiness, low blood pressure, dry mouth, blurred vision, constipation, weight gain, difficulty urinating, and stiffness. In general, Prolixin is less sedating than many other antipsychotics but has more movement side-effects. (Additional information: A rare, but very serious side-effect of all the antipsychotic drugs that you should be aware of is “neuroleptic malignant syndrome.” This condition includes a rapid onset of motor, mental, and autonomic symptoms, such as muscle rigidity, tachycardia, hyperthermia, and altered consciousness. To prevent a potentially fatal outcome, the drug must be stopped as soon as symptoms of NMS develop.)

150. D, Which one of these you would do depends on the situation. Know the limits of your training and experience and when you are not certain seek consultation.
Choice A is incorrect, refer and consult when in doubt of your competence. Maintain your competency by keeping abreast of developments in diagnosis and treatment. Choice B  is incorrect, terminate treatment carefully and for the right reasons. Find an appropriate referral. "Under managed care plans, therapists may be accused of abandonment if they terminate when doing so could result in harm to the client." Choice C is incorrect, practice well within the scope of your competency and expertise.


151. B This item deals with the issues of aging. You need to know how depression masks itself with different age groups.


152. The best approach is to refer him to another therapist or program that specializes in smoking cessation while he continues his anger management work with you.

153.
B
The ego may resort to one of its defense mechanisms when it is unable to ward off danger through using rational, realistic means. Displacement involves the transfer of an instinctual drive from its original target (the boss, in this case) to a less threatening target (the wife) so that the drive can be expressed more safely. Projection involves attributing one’s own unacceptable instinctual needs and drives to someone else; sublimation is a type of displacement in which an unacceptable impulse is diverted into a socially acceptable, even admirable activity; and reaction formation involves avoiding an anxiety-evoking instinct by expressing its opposite.

154.
D
The only reasonable assumption that we can make, based on the information in the question, is that the boy’s lack of interest in learning more about his family’s background is related to his developmental stage. Adolescents are striving to disengage from their families, to attain greater independence, and to develop an identity of their own. On a day-to-day basis, peers play a more dominant role in their lives than family.

155.
A
Body dysmorphic disorder involves preoccupation with an imagined defect in appearance or excessive preoccupation with an existing minor abnormality in appearance.

156.
C
This situation reflects a dual relationship, and the NASW Code of Ethics prohibits social workers from engaging in a dual relationship with a client or former client when there is a risk of exploitation or potential harm to the client. So, the underlying issue here is whether this situation (essentially, an existing dual relationship) poses a risk of exploitation or harm to this client. As the test-taker, you have few details about the situation – you don’t know how big this church is; how much contact individual church members have with each other (before now, however, the social worker didn’t know that he and the client both attend this church, which implies that they’ve never run into each other at church); or whether there is even another church in the community for the social worker to attend (“a”). In other words, this situation is too ambiguous to be a clear example of an unethical (harmful) dual relationship, but it could pose problems in the future. All of this reasoning indicates that the social worker’s “best” action at this time is seek consultation (or supervision) to help him decide how to handle this situation. The Code of Ethics says that if a dual or multiple relationship is unavoidable, a social worker should take steps to protect the client and is responsible for setting clear, appropriate, and culturally sensitive boundaries. Therefore, it would be a good idea for this social worker to discuss with the consultant, among other things, the best way to handle any future encounters with the client at church.


157.
D
The question suggests that the client’s religious faith is important to him (he’s been an active member of his church for many years), but his faith has been challenged by this experience. Given this, the “best” intervention would be to provide the client with a way of finding out that religious support is available to him elsewhere. The referral in “d” offers the client an alternative place to find the religious support he has lost at his church. Some churches (e.g., Fellowship United Methodist Church) are accepting of homosexuals. Moreover, social workers working with gay clients should be aware of and use community resources to enhance their clients’ support network. Option “a” doesn’t address the client’s presenting problem (i.e., he feels alienated at his church, and this is making him question his faith). Another reason why “a” is poor is that it implies that the social worker would focus on questioning the client’s decision-making process, or on the idea that his decision to come out at church was a bad one; whether or not the client’s decision was appropriate, such an intervention would blame the client for what happened at his church after he came out. Option “b” is OK (particularly because it mentions validating the client’s feelings), but the correct answer is better because it offers the client a specific place where he can go to find the support he has lost. And exploring the client’s diminished faith (“c”) is not bad, but the social worker already knows why the client’s faith has been challenged; the correct answer recommends a way of addressing this problem in a direct and immediate way.

158.
B
Mediation is a method of conflict resolution in which a compromise between the opposing parties is sought. The goal of the mediator is to get the two parties to agree on a resolution of the conflict; thus, he or she would encourage the parties to be more flexible about their goals. The mediator uses various tactics and encourages the parties to speak freely in order to facilitate voluntary agreement between the parties. The mediator can make recommendations or suggestions, but he or she has no formal power and cannot impose a solution or decision (which rules out answer “c”). If the parties can’t come to an agreement, then the mediator’s work is finished. When an impasse is reached and a more formal evaluation or recommendation is needed (e.g., when a court is seeking an evaluation or recommendation), this role must be filled by someone other than the mediator. Contrary to choice “a,” the mediator should remain neutral. And choice “d” sounds more like the goal of a therapist than a mediator..

159..
C
Regardless of a client’s cultural background or the possibility that she is having acculturation problems, the first step when assessing a suspected substance use disorder is taking a complete alcohol/drug use history. This information is needed before the social worker can determine appropriate treatment recommendations, including the appropriate level of care (“d”). Moreover, taking this history could help the social worker determine whether this client’s pattern of drug use is related in some way to acculturation problems (“b”).

160.
D Depending on the nature of the violation, you cannot only lose your license, but there can be grounds for legal/criminal action as well.  Choice A is not the best response, Sanctions may include the suspension or revocation of your license to practice. Choice B, C are incorrect.


161.
B
The question emphasizes only the client’s desire to prepare a will. Thus, the social worker should refer the client to an attorney for assistance in writing her will. It would be tempting to explore how the woman is coping with the news that she is terminally ill, but this is not the reason why she was referred to the social worker.

162.
C
Only “c” describes a goal of TA. The goals of TA are to help the client make new decisions about her life that reflect the following: an integration of the three ego states – adult, parent, and child (which both rules out “a” and shows that “c” is correct); an “I’m OK/You’re OK” life position; and flexible, autonomous (“scriptless”) behavior (which rules out “b”). Answer “d” is wrong for two reasons: First, the adoption of a “success identity” is a goal of Glasser’s reality therapy. Second, TA therapists attempt to help clients stop playing “games” – they don’t help them learn more “more adaptive” games. “Games” are repeated ulterior transactions (i.e., transactions involving two ego states in the initiator and/or responder and a disguised message) that appear to provide strokes and promote intimacy, but, in fact, allow people to avoid getting close and advance their life scripts.


163.
D
It was important to note this client’s age – 18. In addition, the question provides no information about what arrangements the social worker has made with the client and her parents regarding sharing confidential information (we don’t know whether the social worker has a release from the client allowing her speak with the parents). Therefore, “d” is correct. What may have misled you in this case, however, is that the client’s parents are paying for her therapy. Parents or others paying for an adult client’s treatment do sometimes assume that they have a right to be given confidential information about the client, but this is not true. A social worker should make information available to the people paying for a client’s therapy only at the client’s request or when the client has consented to have the information released at the request of another person, and only when disclosing the information is in the client’s best interests. At the beginning of therapy, a social worker should clarify with all involved persons (in this case, the client and her parents) the conditions surrounding the release of confidential information.


164. A This question may be confusing because it contains a double negative. You can change a double negative to a positive – for this question, that means you are looking for something suggesting that this client is a good candidate for group therapy. Contraindications for group therapy include: Low motivation to change (“b”); certain symptoms, diagnoses, or characteristics – e.g., severe depression (“c”) and withdrawal, paranoia, acute psychosis, brain damage, sociopathy, a high level of denial, high somatization, low self-esteem (“d”), low psychological sophistication, and low IQ; incompatibility with group norms for acceptable behavior (e.g., significant deviancy from the other group members); inability to tolerate the group setting; unfavorable expectations about group treatment; and acute crisis. By contrast, “a” would indicate that this client may be a good candidate for group therapy: Groups are often able to confront resistance more quickly and effectively than a single therapist.


165.A   According to psychoanalytic theory, a patient with an obsessive-compulsive neurosis is likely to strongly rely on which of the following sets of defenses?

166.
C
A social learning approach draws concepts and techniques from behavioral therapy. With families, therapists using a social learning approach assume that family members have failed to learn basic relationship, communication, and conflict-resolution skills; consequently, treatment focuses on teaching these skills. The behavioral technique “time-out” is often taught to parents as an alternative to physical discipline. Time-out involves removing a child from an environment that is reinforcing an unwanted behavior for specific period of time, with the goal of reducing or eliminating that behavior.


167.
A
This question is difficult because when using play techniques, therapists generally do not direct a child’s play behaviors or statements; instead, they allow the child to “lead the way” (V.M. Axline, “Play Therapy,” Ballantine Books, New York, p. 119, 1947). However, of the answers offered, “a” is the best one. The action in “a” would allow the social worker to check out the validity of her belief, and also provide the boy with a therapeutic opportunity to express his feelings verbally. Answer “b” is too directive for play therapy. It is also a roundabout and probably ineffective way of finding out what the boy is feeling. It seems to assume that he will play out another angry scene and that, if he does, this will somehow give the social worker more insight than the first game did. In other words, this action fails to capitalize directly on an opportunity that is already present, the chance to encourage the boy to talk about his first game. An interpretation at this point (“c”) would probably be less beneficial than providing the boy with an opportunity to verbalize his own feelings. The social worker could help the boy label his emotions after he has expressed them, if he needed help with this, but she first should provide him with an opportunity to express himself. Rule out “d” because it’s better for the social worker to give the boy a chance to identify and verbalize his feelings on his own, rather than presuming that she knows what he feels. He may clam up or just agree with the social worker, no matter what he truly feels, if she puts words in his mouth.


168.
B
The best answer offered is “b” because the NASW Code of Ethics requires social workers to consult and “cooperate with other professionals in order to serve their patients or clients effectively and appropriately.” Moreover, a social worker is not qualified to decide whether a client should or should not take a medication. Instead, an appropriate role for a social worker to take in this situation is to (with the client’s permission) discuss the client’s needs and concerns with the doctor, including his psychosocial needs. Answer “c” is true but it inappropriately dismisses the client’s concerns, which seem legitimate – i.e., it would be irresponsible to assume that a client who has past problems with alcohol use and a family history of drug abuse will have no problem taking this pain medication only as prescribed. Finally, behavioral and cognitive-behavioral interventions (“d”) have been found effective for treating many forms of chronic pain, but, again, it would unethical and illegal for a social worker to dismiss or override a physician’s recommendations regarding medication for a client.


169..
C
Psychoanalytic theory suggests that the paranoid individual “projects” his or her own threatening or unacceptable impulses onto others. Freud suggested that reaction formation (“a”) underlies obsessive-compulsive personality traits. He viewed certain obsessive-compulsive personality traits as obedience in defense against unacceptable rageful impulses. Freudian personality theory proposes that introjection (“b”), as well as regression, underlies depression. For example, the person internalizes anger he or she feels toward someone else. And Freud believed that displacement (“d”) underlies obsessions.


170.
A
This question doesn’t specify why the attorney is requesting the records, nor does it indicate whom the attorney represents. An ethical social worker, however, would not simply release a client’s records based on just a request from an attorney (“b”), without having prior permission from the client to do so. Instead, a social worker would take steps to protect the client’s confidentiality to the greatest extent permitted by law. So, while this question is vague, we can assume that the test-writers are assessing whether you know about this obligation. With this in mind, your first reaction in this situation should be to assert the privilege on your client’s behalf and refuse the attorney’s request for now. You might ultimately advise the client to sign a release of information for the records (once you have more information about the purpose of the request), but simply having him or her sign a release now (“c”) would not be a good first step: Instead, you will need to discuss the attorney’s request with the client and, if relevant, with the client’s attorney; in these discussions, you will want to go over the possible consequences of releasing the records to the attorney. If the client did end up signing a release, you could then provide the records. On the other hand, if the client refused to sign the release after you discussed the matter with him or her, you would need to continue asserting the privilege. The issue could then end up in court: If the court decided that the privilege should be waived in this case, it would issue an order requiring you to release the client’s records; you would then have to comply with the order or else risk a charge of court contempt. “D” is incorrect because you would violate the client’s confidentiality if you had any additional discussion with this attorney about the client or the request for his or her records before speaking with your client and getting a signed release form.


 ALL THE BEST & GOD BLESS !!!